Kaplan Physiology Flashcards

1
Q
  1. Serum chemistry studies of a patient reveal that her aspartate aminotransferase (AST) is
    markedly elevated, whereas her alanine aminotransferase (ALT), gamma-glutamyl
    transpeptidase (GGT), and alkaline phosphatase are all within normal limits. Disease of which
    of the following organs would be most likely to cause this serum enzyme pattern?
    A. Colon
    B. Duodenum
    C. Heart
    D. Pancreas
    E. Stomach
A

The correct answer is C. Myocardial infarction (MI) can cause AST elevation without accompanying
elevation of ALT or other liver enzymes. This is an important fact to remember because it may be the
first clue for heart disease in a patient who has an atypical presentation of MI (as is common in
women with MI). MI can be confirmed with measurement of the MB fraction of creatine phosphokinase
(CPK-MB). In addition, Troponin T and Troponin I can be diagnostic.
Unfortunately, diseases of the tubular organs of the gastrointestinal tract, including colon (choice A),
duodenum (choice B), and stomach (choice E), do not produce distinctive serum enzyme patterns.
Damage to the pancreas (choice D) is associated with elevated amylase levels.

How well did you know this?
1
Not at all
2
3
4
5
Perfectly
2
Q
  1. A newborn infant has multiple hemorrhages. Clotting studies demonstrate an elevated
    prothrombin time and elevated INR. An abnormality of which of the following biochemical
    processes is likely present in this patient?
    A. Conversion of homocysteine to methionine
    B. Conversion of methylmalonyl CoA to succinyl CoA
    C. Degradation of cystathionine
    D. Formation of gamma-carboxyglutamate residues
    E. Hydroxylation of proline
A

The correct answer is D. Deficiency of vitamin K produces a clotting disorder characterized by an
elevated prothrombin time and easy bleeding, particularly in neonates (hemorrhagic disease of the
newborn). The biochemical basis for this hemorrhagic tendency is that glutamate residues on Factors
II (Thrombin), VII, IX, and X must be converted to gamma-carboxyglutamate residues (in a vitamin Krequiring
reaction) for optimal activity.
The conversion of homocysteine to methionine (choice A) requires vitamin B12. Vitamin B12
deficiency can result in the development of pernicious anemia. Conversion of methylmalonyl CoA to
succinyl CoA (choice B) requires vitamin B12.
Degradation of cystathionine (choice C) requires vitamin B6 and can result in the development of
mouth soreness, glossitis, cheilosis, and weakness.
Hydroxylation of proline (choice E) requires vitamin C. Vitamin C deficiency can cause easy bruising,
but will not prolong the prothrombin time. Vitamin C deficiency is commonly known as scurvy.

How well did you know this?
1
Not at all
2
3
4
5
Perfectly
3
Q
3. Where are the body's temperature regulation centers located?
A. Midbrain
B. Pons
C. Medulla
D. Hypothalamus
A

The correct answer is D. Temperature regulation centers and thirst and food intake regulatory
centers are located in the hypothalamus. Choice A - Midbrain contains the micturation center. Choice
2
B - Pneumotaxic centers are located in the pons. Choice C - The medulla contains the vasomotor
center, respiratory center, vomiting, swallowing, and coughing centers.

How well did you know this?
1
Not at all
2
3
4
5
Perfectly
4
Q
4. An animal is made diabetic by injection of a drug that destroys pancreatic ß cells. Removal
of which of the following organs would most likely produce a decrease in blood glucose
concentration in this animal?
A. Anterior pituitary
B. Colon
C. Gonads
D. Kidney
E. Pancreas
A

The correct answer is A. The anterior pituitary produces the following hormones: thyroid-stimulating
hormone (TSH), adrenocorticotropic hormone (ACTH), follicle stimulating hormone (FSH), leutinizing
hormone (LH), prolactin, and growth hormone (GH). Two of these secretions (GH, ACTH) affect the
sensitivity of peripheral tissues to the action of insulin. Growth hormone has a direct effect on liver and
muscle to decrease insulin sensitivity. This may be partly through a growth hormone-induced decline
in insulin receptors or to unknown postreceptor defects. In excess, growth hormone is “diabetogenic,”
and approximately 25% of patients with acromegaly have diabetes. ACTH indirectly has antiinsulin
effects by virtue of the cortisol secretion it evokes. Like growth hormone, cortisol also decreases
insulin sensitivity in peripheral tissues. A third anterior pituitary hormone, TSH, also tends to increase
blood glucose levels. In this case, the effect is probably mediated mostly through increased glucose
absorption by the gut. Patients with hyperthyroidism can sometimes exhibit a postprandial glucosuria
because of excessive intestinal glucose absorption. In diabetic animals, the removal of the anterior
pituitary may lower blood glucose by increasing tissue sensitivity to whatever insulin remains.
Removal of the colon (choice B) should have little effect on blood glucose because dietary glucose is
absorbed in the small intestine.
Sex steroids secreted by the gonads (choice C) have little effect on blood glucose concentration.
The kidney (choice D) plays an important role in reabsorbing filtered glucose. In diabetes, the tubular
reabsorption maximum is exceeded and glucose spills over into the urine. The loss of glucose in the
urine helps to reduce the severity of the plasma hyperglycemia. Removal of the kidneys would, if
anything, make the hyperglycemia worse. Pancreatectomy (choice E) would make the hyperglycemia
worse by removing the source of any remaining insulin.

How well did you know this?
1
Not at all
2
3
4
5
Perfectly
5
Q
5. Which of the following is most likely to decrease in the skeletal muscles of a healthy 22-
year-old woman during exercise?
A. Arteriolar resistance
B. Carbon dioxide concentration
C. Lactic acid concentration
D. Sympathetic nervous activity
E. Vascular conductance
A

The correct answer is A. The increase in muscle blood flow that occurs during exercise is caused
by dilation of the arterioles (i.e., decreased arteriolar resistance) attributed to the dilatory actions of
metabolic factors (e.g., adenosine, lactic acid, carbon dioxide) produced by the exercising muscles. In
normal skeletal muscles, the blood flow can increase as much as 20-fold during strenuous exercise.
Exercise causes the concentration of carbon dioxide (choice B) and lactic acid (choice C) to increase
in the muscles.
Mass discharge of the sympathetic nervous system (choice D) occurs throughout the body during
exercise, causing arterioles to constrict in most tissues. The arterioles in the exercising muscles,
however, are strongly dilated by vasodilator substances released from the muscles.
3
A decrease in vascular conductance (choice E) occurs when the vasculature is constricted.
Resistance and conductance are inversely related, so that a decrease in arteriolar resistance is
associated with an increase in arteriolar conductance.

How well did you know this?
1
Not at all
2
3
4
5
Perfectly
6
Q
6. Which of the following parameters is expected to increase in response to a 50% reduction in
sodium intake for a 2-month period?
A. Arterial pressure
B. Atrial natriuretic peptide release
C. Extracellular fluid volume
D. Renin release
E. Sodium excretion
A

The correct answer is D. Renin is an enzyme released by the juxtaglomerular cells when renal blood
pressure or pO2 declines. Renin converts angiotensinogen to angiotensin I. A reduction in sodium
intake leads to a decrease in extracellular fluid volume (choice C) and therefore a decrease in arterial
pressure (choice A). The decrease in arterial pressure stimulates renin release that in turn leads to
an increase in the formation of angiotensin II. The angiotensin II increases the renal retention of salt
and water (i.e., decreases sodium excretion, choice E), which returns the extracellular fluid volume
nearly back to normal.
Atrial natriuretic peptide (choice B) is released from the two atria of the heart as a result of an
increase in the extracellular fluid volume. A decrease in sodium intake therefore would tend to
decrease the release of atrial natriuretic peptide.

How well did you know this?
1
Not at all
2
3
4
5
Perfectly
7
Q
7. Which of the following hormones is most important in the initiation of gallbladder
contraction following a fatty meal?
A. CCK
B. Gastrin
C. GIP
D. Secretin
E. VIP
A

The correct answer is A. Cholecystokinin, or CCK, is synthesized in the duodenal and jejunal
mucosa and stimulates gallbladder contraction and pancreatic enzyme secretion. Other functions
include slowing of gastric emptying, an atrophic effect on the pancreas, and secretion of antral
somatostatin, which in turn decreases gastric acid secretion.
Gastrin (choice B) prepares the stomach and small intestine for food processing, including
stimulating secretion of HCl, histamine, and pepsinogen, increasing gastric blood flow, lower
esophageal sphincter tone, and gastric contractions. Gastric inhibitory peptide, or GIP (choice C),
stimulates pancreatic insulin secretion at physiologic doses and inhibits gastric acid secretion and
gastric motility at pharmacologic doses. Secretin (choice D) stimulates secretion of bicarbonatecontaining
fluid from the pancreas and biliary ducts. Vasoactive intestinal polypeptide, or VIP (choice
E), relaxes intestinal smooth muscle and stimulates gut secretion of water and electrolytes.

How well did you know this?
1
Not at all
2
3
4
5
Perfectly
8
Q
8. During spermatogenesis, crossing over occurs during the meiotic division of which of the
following cells?
A. Primary spermatocytes
B. Secondary spermatocytes
C. Spermatids
D. Spermatogonia
E. Spermatozoa
A

The correct answer is A. Crossing over, a transposition of genetic information, occurs during the
first meiotic (reduction) division, when the primary spermatocyte divides to form two secondary
4
spermatocytes. This division does not consist of separation of sister chromatids after DNA replication,
but rather involves the separation of previously paired, homologous chromosomes. Crossing over
occurs during prophase of meiosis I.
The secondary spermatocyte (choice B) undergoes the second meiotic division, which results in four
spermatids (choice C), each with the haploid number of chromosomes. The spermatids are located
adjacent to the lumen of the seminiferous tubules and are distinguished by their small size. These
cells undergo no further division, but become transformed into mature spermatozoa (choice E)
through the prcess of spermiogenesis.
The spermatogonia (choice D) are the primitive germ cells. These cells give rise to the primary
spermatocytes through repeated mitotic divisions.
Remember the sequence of spermatogenesis:
Spermatogonia (2n) ? REPEATED MITOSES ? Primary spermatocytes (2n) ? FIRST MEIOTIC
DIVISION ? Secondary spermatocytes (n) ? SECOND MEIOTIC DIVISION ? Spermatids (n) ?
SPERMIOGENESIS ? Spermatozoa (n)

How well did you know this?
1
Not at all
2
3
4
5
Perfectly
9
Q
  1. A decrease in which of the following is the most likely cause of peripheral edema in a
    patient with long-term alcoholism and liver disease?
    A. Capillary hydrostatic pressure
    B. Interstitial colloid osmotic pressure
    C. Interstitial hydrostatic pressure
    D. Plasma colloid osmotic pressure
    E. Precapillary arteriolar resistance
A

The correct answer is D. Osmotic pressure of a solution is an indicator of the force of water
movement resulting from its solute concentration. The higher the solute concentration of a solution,
the greater its osmotic pressure. The plasma colloid osmotic pressure is often low in alcoholics with
chronic liver disease (cirrhosis). The diseased liver cannot produce adequate amounts of albumin,
which leads to a decrease in the concentration of albumin in the plasma (i.e., hypoalbuminemia).
Because approximately 75% of the plasma colloid osmotic pressure can be attributed to the presence
of albumin in the plasma, the decrease in plasma albumin concentration that occurs in the latter
stages of cirrhosis often leads to peripheral edema. Cirrhosis also causes excess fluid to accumulate
in the peritoneal cavity as ascites. In the case of ascites, the edema results not only from
hypoalbuminemia, but also from portal vein obstruction (which increases capillary hydrostatic
pressure) and the obstruction of lymphatic drainage of the liver. In fact, ascites is observed more often
than peripheral edema in liver disease.
A decrease in capillary hydrostatic pressure (choice A) would tend to decrease fluid loss from the
capillaries, and thereby oppose the development of edema. A decrease in the colloid osmotic
pressure of the interstitial fluid (choice B) would decrease fluid loss from the capillaries, thereby
opposing the development of edema. A decrease in interstitial hydrostatic pressure (choice C) would
tend to increase fluid loss from the capillaries, but this cannot be considered a primary cause of
edema because the interstitial hydrostatic pressure actually increases when a tissue becomes
edematous. A decrease in precapillary arteriolar resistance (choice E), which means arteriolar
dilation, would increase capillary hydrostatic pressure and tend to cause edema. Decreased
precapillary arteriolar resistance in the peripheral vasculature, however, is not associated with
cirrhosis.

How well did you know this?
1
Not at all
2
3
4
5
Perfectly
10
Q
  1. The medical record of a patient indicates a systolic murmur due to increase in afterload,
    producing a pressure gradient between the ventricle and aorta during ejection. Which of the
    following best describes the condition?
    A. Aortic insufficiency
    B. Aortic stenosis
    C. Mitral insufficiency
    D. Mitral stenosis
A

The correct answer is B.Aortic stenosis increases afterload and produces a pressure gradient
between ventricle and aorta during ejection. Aortic insufficiency (choice A) increases preload and
produces retrograde flow from the aorta to the ventricle, leading to a diastolic murmur. Mitral
insufficiency (choice C) increases volume and pressure in the atrium and ventricle, producing a
systolic murmur. Mitral stenosis (choice D) increases left-atrial volume and pressure, producing a
diastolic murmur.

How well did you know this?
1
Not at all
2
3
4
5
Perfectly
11
Q
11. At 25 weeks of pregnancy, an unidentified infection greatly compromises the viability of a
developing fetus. The level of which of the following hormones in the mother's blood is most
likely to be affected?
A. Estriol
B. Free thyroxine
C. Human chorionic gonadotropin
D. Human chorionic somatomammotropin
E. Progesterone
A

The correct answer is A. Plasma levels of maternal estrogens during pregnancy depend on a
functioning fetus. The fetal adrenal cortex and liver produce the weak androgens, DHEA-S and 16-OH
DHEA-S, which are carried to the placenta by the fetal circulation. The placenta then desulfates the
androgens and aromatizes them to estrogens (16-OH DHEA-S, estriol) before delivery to the maternal
circulation. Estradiol and estrone increase approximately 50-fold during pregnancy, but estriol
increases approximately 1,000 fold. When estriol is assayed daily, a significant drop may be a
sensitive early indicator of fetal jeopardy. Total serum thyroxine concentration may be increased in
pregnancy because of an increase in circulating TBG resulting from increased estrogen. Free
thyroxine, however, (choice B) remains within the normal range because of feedback regulation. The
decline in estrogen with fetal compromise may gradually decrease serum thyroxine, but the free
thyroxine will remain unchanged. Human chorionic gonadotropin (choice C) and human chorionic
somatomammotropin (choice D) are both secreted by syncytiotrophoblasts of the placenta. As long
as placental function is intact, blood levels of these two hormones should not change with fetal
compromise. Placental secretion of progesterone (choice E) during pregnancy is also independent of
any fetal contribution. The placenta relies on maternal cholesterol for progesterone production. Fetal
death has no immediate influence on progesterone production by the placenta.

How well did you know this?
1
Not at all
2
3
4
5
Perfectly
12
Q
  1. A healthy 20-year-old man deprived of water for several days has a plasma concentration
    of antidiuretic hormone (ADH) 5 times greater than normal. Which of the following is the most
    likely explanation for the increase in ADH concentration?
    A. Decreased plasma aldosterone
    B. Decreased plasma renin activity
    C. Increased extracellular fluid volume
    D. Increased left atrial pressure
    E. Increased plasma osmolality
A

The correct answer is E. An obligatory loss of water from the body continues to occur even when a
person is deprived of water. This loss of water from the body tends to concentrate the extracellular
fluid, causing it to become hypertonic. Both the decrease in extracellular fluid (compare with choice
C) and the increase in osmolarity act as stimuli for increased thirst and increased secretion of ADH.
The decrease in extracellular fluid volume also tends to decrease arterial pressure that in turn
increases plasma renin activity (compare with choice B) and aldosterone levels in the plasma
(compare with choice A). Water deprivation tends to decrease left atrial pressure (compare with
choice D).

How well did you know this?
1
Not at all
2
3
4
5
Perfectly
13
Q
  1. Which of the following indices would be expected to be decreased in a kidney donor after
    full recovery from the removal of the kidney?
    A. Creatinine clearance
    B. Creatinine production
    C. Daily excretion of sodium
    D. Plasma creatinine concentration
    E. Renal excretion of creatinine
A

The correct answer is A.Because creatinine is freely filtered by the glomerulus but not secreted or
reabsorbed to a significant extent, the renal clearance of creatinine is approximately equal to the
glomerular filtration rate. In fact, creatinine clearance is commonly used to assess renal function in the
clinical setting. When a kidney is removed, the total glomerular filtration rate decreases because 50%
of the nephrons have been removed, which causes the creatinine clearance to decrease. In turn, the
plasma creatinine concentration (choice D) increases until the rate of creatinine excretion by the
kidneys (choice E) is equal to the rate of creatinine production by the body. Recall that creatinine
excretion = GFR x plasma creatinine concentration. Creatinine excretion is therefore normal when
GFR is decreased following removal of a kidney because the plasma concentration of creatinine is
elevated. Creatinine is a waste product of metabolism. Creatinine production (choice B) is directly
related to the muscle mass of an individual, but is independent of renal function. The daily excretion of
sodium (choice C) is unaffected by the removal of a kidney. The amount of sodium excreted each
day by the remaining kidney exactly matches the amount of sodium entering the body in the diet.

How well did you know this?
1
Not at all
2
3
4
5
Perfectly
14
Q
14. An elevated level of which of the following substances would identify a specimen as
plasma rather than serum?
A. Albumin
B. Erythrocytes
C. Fibrinogen
D. Granulocytes
E. Serotonin
A

The correct answer is C. This is simply a definition What is the difference between serum and
plasma? Essentially, serum is derived from plasma by the extraction of fibrinogen and coagulation
factors II, V, and VIII. This can be achieved by allowing whole blood to clot, then removing the clot.
Albumin (choice A) is present in serum and plasma. Neither erythrocytes (choice B) nor
granulocytes (choice D) are present in either serum or plasma. Serotonin (choice E) levels may be
increased in serum because of the platelet breakdown that occurs during the extraction process.
Serotonin is normally found in the highest concentration in platelets, and in the enterochromaffin cells
and myenteric plexus of the gastrointestinal tract. The brain and the retina contain smaller amounts.

How well did you know this?
1
Not at all
2
3
4
5
Perfectly
15
Q
  1. A dietary deficiency of which of the following is most strongly associated with a neural
    tube defect at the base of the fetus’s spine?
    A. Calcium
    B. Folate
    C. Iron
    D. Vitamin C
    E. Vitamin K
A
The correct answer is B. Neural tube defects most commonly occur in the lumbosacral region,
typically resulting in motor and sensory deficits in the lower extremities, and bowel and bladder
dysfunction. This condition is now known to be associated with low maternal folate during the first 3-4
weeks of pregnancy, a time when many women may be unaware of their pregnancy. It is now
recommended that all women of childbearing age consume at least 400 grams of folic acid daily.
Calcium deficiency (choice A) can cause osteoporosis and osteopenia. Iron deficiency (choice C)
can cause iron deficiency anemia. Vitamin C deficiency (choice D) can cause scurvy. Vitamin K
deficiency (choice E) can cause a bleeding diathesis.
How well did you know this?
1
Not at all
2
3
4
5
Perfectly
16
Q
16. Which of the following substances can be converted to glucose to supply the needs of the
brain during starvation?
A. Acetoacetate
B. Acetone
C. Amino acids
D. Beta-hydroxybutyrate
E. Fatty acids
A

The correct answer is C. During starvation, the diet is inadequate to provide sufficient glucose to
maintain the brain, yet the brain requires glucose as an energy source. Glucose used in the brain
during starvation is synthesized from amino acids, primarily derived from muscle protein. This use of
amino acids in starvation leads to profound muscle wasting.
The ketone bodies (acetoacetate, choice A; acetone, choice B; and beta-hydroxybutyrate, choice D)
produced during starvation and diabetic ketoacidosis are derived from adipose triacylglycerols.
Although these compounds can be used in biochemical pathways in the brain, they cannot completely
replace glucose in that organ. Furthermore, glucose cannot be synthesized from these precursors.
Fatty acid (choice E) degradation cannot be used to produce glucose. It can be used, however, to
produce ketone bodies that can be used by the brain as a source of intermediates for some synthetic
pathways.

How well did you know this?
1
Not at all
2
3
4
5
Perfectly
17
Q
17. Absence of which of the following cell types is responsible for loss of vitamin B12
absorption?
A. Chief cells
B. G cells
C. Goblet cells
D. Mucous neck cells
E. Parietal cells
A

The correct answer is E. The parietal cells of the stomach produce intrinsic factor, a glycoprotein
that binds vitamin B12 in the lumen of the stomach and facilitates its absorption in the terminal ileum.
Patients without a stomach and those with pernicious anemia (autoimmune destruction of parietal
cells) require B12 replacement therapy. Recall that B12 deficiency will lead to megaloblastic anemia
and a blood smear with hypersegmented neutrophils. Note that parietal cells also synthesize and
secrete HCl. Chief cells (choice A) are responsible for secreting pepsinogen, the precursor to pepsin.
G cells (choice B) secrete gastrin, which stimulates secretion of acid by the parietal cells found in the
body and fundus of the stomach. Zollinger-Ellison syndrome is caused by a pancreatic or duodenal
tumor that secretes gastrin (a gastrinoma). It is characterized by the development of severe peptic
ulcer disease. Goblet cells (choice C) are part of the mucosa of the small intestine, not the stomach.
They produce glycoproteins (mucins) that protect and lubricate the lining of the intestine. Mucous
neck cells (choice D) secrete mucus and are located in the necks of the gastric glands.

How well did you know this?
1
Not at all
2
3
4
5
Perfectly
18
Q
  1. Which of the following is most likely to relieve the symptoms caused by hyperventilation?
    A. Breathing a 10% oxygen/90% nitrogen mixture
    B. Breathing 100% nitrogen
    C. Breathing in and out of a plastic bag
    D. Intravenous administration of bicarbonate
    E. Lying down
A

The correct answer is C. Hyperventilation os often associated with states of anxiety and can lead to
feelings of faintness, suffocation, tightness in the chest, and blurred vision. Individuals undergoing
such an attack may not be aware of overbreathing. The anxious, hyperventilating patient is “blowing
off” carbon dioxide, which lowers the arterial PCO2. Many of the symptoms associated with anxiety
attacks are probably caused by a decrease in cerebral blood flow secondary to low arterial PCO2.
Recall that carbon dioxide is a major regulator of cerebral blood flow, i.e., carbon dioxide dilates the
brain vasculature, and conversely, the vasculature constricts when carbon dioxide levels are low. The
decrease in cerebral blood flow leads to cerebral hypoxia, which is probably responsible for the
fainting and blurred vision. An attack may be terminated by breathing in and out of a plastic bag
because this can increase carbon dioxide levels in the blood. Inhaling a 5% carbon dioxide mixture
would also be effective. Breathing a mixture of 10% oxygen/90% nitrogen (choice A) or 100%
nitrogen (choice B) can decrease oxygen delivery to the brain and thereby worsen the symptoms
caused by hyperventilation. Hyperventilation results in hypocapnia (low PCO2), which causes
alkalosis (high blood pH). Bicarbonate (choice D) should not be administered to an alkalotic patient.
8
The feelings of faintness and blurred vision resulting from hyperventilation are not relieved by lying
down (choice E).

How well did you know this?
1
Not at all
2
3
4
5
Perfectly
19
Q
  1. A student volunteers to have his lung volumes and capacities measured. He is instructed
    to breathe several times until the helium has equilibrated between the spirometer and his
    lungs, and then exhale as much air as he possibly can. Calculations are made to determine the
    amount of air remaining in his lungs, which is called the
    A. expiratory reserve volume
    B. functional residual capacity
    C. inspiratory capacity
    D. inspiratory reserve volume
    E. residual volume
A

The correct answer is E. There are two ways to arrive at the correct answer to this question. The
first is to simply remember the definition of residual volume (RV): the amount of air remaining in the
lungs after maximal exhalation. The second way is to recall that the helium dilution technique
described above is used to measure functional residual capacity (FRC) and RV, which narrows the
reasonable option choices to B and E. All of the other volumes and capacities can be directly
measured with spirometry because they are blown into the spirometer. Only FRC and RV represent
amounts of air that remain in the lungs. Expiratory reserve volume (choice A) is the volume expelled
by an active expiratory effort after passive expiration. Functional residual capacity (choice B) is
defined as the amount of air remaining in the lungs after passive expiration. Inspiratory capacity
(choice C) is the maximal amount of air inspired after a passive expiration. Inspiratory reserve volume
(choice D) is the amount of air inspired with a maximal inspiratory effort over and above the tidal
volume.

How well did you know this?
1
Not at all
2
3
4
5
Perfectly
20
Q
  1. Which of the following characteristics is similar for spermatogenesis and oogenesis?
    A. Age at which meiosis begins
    B. Amount of cytoplasm retained
    C. DNA replication during meiosis
    D. Length of prophase I
    E. Transmission to fetus of mitochondrial DNA
A

The correct answer is C. The actual process of manipulation of DNA and chromosomes during
meiosis is very similar in spermatogenesis and oogenesis. The processes also differ in many other
respects, however:
In oogenesis, the process of meiosis begins before birth and arrests between birth and puberty in
prophase I. In contrast, spermatogenesis does not begin until puberty (choice A).
The egg retains a large volume of cytoplasm (choice B), where nearly all the cytoplasm is stripped
during formation of a sperm. As noted, in oogenesis, meiosis is arrested in prophase I, which is
consequently very prolonged in the female. In spermatogenesis, meiosis is completed in a much
shorter time (choice D). Both the egg and the sperm have mitochondria, but those of the sperm are
left outside when the sperm nucleus enters the egg and consequently do not contribute to the
mitochondrial genome of the fetus. Instead, the mitochondria are transmitted from the egg to the fetus
(choice E). Traits coded for by mitochondrial DNA are therefore inherited in a matrilineal fashion.

How well did you know this?
1
Not at all
2
3
4
5
Perfectly
21
Q
  1. Increases in which of the following normal physiological parameters may have contributed
    to the formation of acute gastric stress ulcers?
    A. Bicarbonate transport
    B. Epithelial regenerative capacity
    C. Mucosal blood flow
    D. Mucus secretion
    E. Pepsin production
A

The correct answer is E. Pepsin is produced in the chief cells in the stomach. This proteolytic
enzyme functions most effectively at a strong acid pH of 1.5 - 2.0. Pepsin production is a normal
physiologic activity of the stomach that, in conditions of stress, may overwhelm the stomach’s
weakened defenses and result in gastric ulceration. Gastric acid production is another condition that
may increase and cause acute ulceration. Furthermore, these two factors may remain unchanged and
still result in gastric ulcers if the gastric defenses are weakened by stress. All of the other choices
represent normal defensive forces in the stomach.
Increased bicarbonate transport (choice A) would protect the gastric epithelium from the potentially
harmful acidity of the gastric contents. The adherent mucus is relatively alkaline, providing local
protection to the superficial mucosa.
Gastric epithelial cells can normally replicate rapidly, allowing mucosal defects to be rapidly repaired.
Increasing the regenerative capacity of the epithelium (choice B) would have a protective effect
against ulceration.
The gastric mucosa is richly supplied with blood, providing the epithelial cells with an ample supply of
nutrients, oxygen, and bicarbonate to contend with the harsh gastric microenvironment. Stress ulcers
are associated with compromised gastric blood flow, not increased flow (choice C).
Mucus protects the gastric epithelium by virtue of being water insoluble, impermeable to pepsin, and
slowly permeated by acid (H+). Increasing mucus production (choice D) has a protective effect for the
gastric mucosa.

How well did you know this?
1
Not at all
2
3
4
5
Perfectly
22
Q
22. Inhibition of which of the following could be responsible for impotence as a side effect of a
medication?
A. Conversion of DHT to testosterone
B. Forward motility factor
C. Nitric oxide synthase (NOS)
D. Oxytocin
E. Prostaglandins
A

The correct answer is C. Penile erection is mediated by the parasympathetic nervous system. The
neurons involved are termed nonadrenergic, noncholinergic (NANC) autonomic neurons, and they
release nitric oxide (NO). NO binds to the iron in the heme molecule of guanylate cyclase, activating it
to form cGMP. This results in a decrease in intracellular calcium and subsequent smooth muscle
relaxation and vasodilation in the corpus cavernosa, producing erection. Nitric oxide synthase (NOS)
is the enzyme required for the formation of NO from circulating arginine, and androgens are
necessary to maintain normal amounts of this enzyme. Inhibition of this enzyme could result in
impotence. Inhibition of the conversion of testosterone to DHT, not DHT to testosterone (choice A),
would be accomplished by a 5-alpha-reductase inhibitor. This could cause impotence by its
antiandrogen effect. In fact, finasteride, a clinically used 5-alpha-reductase inhibitor, produces drastic
decreases in libido in some men. Inhibition of forward motility factor (choice B) would hamper sperm
motility and result in infertility, not impotence. Inhibition of oxytocin (choice D) or prostaglandins
(choice E) would result in failure to ejaculate (ejaculatory incompetence), not impotence.

How well did you know this?
1
Not at all
2
3
4
5
Perfectly
23
Q
  1. A research physiologist decides to use a marker to measure the volume of total body water
    in a volunteer. Which of the following substances would he most likely use?
    A. Antipyrine
    B. Cresyl violet
    C. Evans blue
    D. I131-albumin
    E. Inulin
A

The correct answer is A. Antipyrine and tritium are both markers for total body water. Cresyl violet
(choice B) is a histologic dye used to stain Missl substance in neurons. It stains cell bodies. Evans
blue (choice C) is used to measure the plasma compartment.
10
I131-albumin (choice D) is used to measure the plasma compartment. Inulin (choice E) is used to
measure the extracellular fluid compartment.

How well did you know this?
1
Not at all
2
3
4
5
Perfectly
24
Q
24. Which of the following vascular structures contains the largest proportion of the total
blood volume in a normal individual?
A. Aorta and large arteries
B. Arterioles
C. Capillaries
D. Chambers of the heart
E. Venules and veins
A

The correct answer is E. The total blood volume of the body is approximately 5000 mL. The
systemic veins contain approximately 64% of this volume or approximately 3200 mL. No other
segment of the circulation comes close to the amount of blood contained by the systemic veins: the
chambers of the heart (choice D) contain approximately 350 mL; the aorta and large arteries (choice
A) together contain approximately 650 mL; and the arterioles and capillaries (choices B and C)
together contain approximately 350 mL. Although the capillaries contain less than 7% of the total
blood volume, they have a very large surface area that facilitates diffusion exchange of nutrients and
metabolites between the blood and tissue spaces.

How well did you know this?
1
Not at all
2
3
4
5
Perfectly
25
Q
  1. Which of the following substances is likely to be present in significant amounts in urine
    that has a stable, frothy, white foam on top?
    A. Bilirubin
    B. Blood
    C. Glucose
    D. Ketones
    E. Protein
A

The correct answer is E. In this case, you need to know that a stable froth on urine is usually caused
by proteinuria (more than several grams per 24 hours). High levels of bilirubin (choice A) in urine can
cause an unusual yellow foam. Blood in the urine (choice B) might be present in some forms of renal
disease, but would not explain the stable foam. High levels of glucose (choice C) in urine can cause it
to develop a sweet smell and taste; smelling and tasting urine was an ancient method of diagnosing
diabetes mellitus, but is no longer recommended. Ketones (choice D) may give urine an acetone-like
odor.

How well did you know this?
1
Not at all
2
3
4
5
Perfectly
26
Q
  1. Tissue taken from a fracture site of a patient with end stage renal failure is noted to have
    increased osteoclastic activity, with notable tunnel-like dissection by osteoclasts into the bony
    trabeculae. Increased levels of which of the following hormones is most likely to be
    responsible for this lesion?
    A. Calcitonin
    B. Cortisol
    C. Erythropoietin
    D. Glucagon
    E. Parathyroid hormone
A

The correct answer is E. The histological picture suggests hyperparathyroidism, a well-recognized
sequela of chronic renal failure. This condition is caused by high levels of serum phosphate and low
serum calcium, which stimulate the release of parathyroid hormone (PTH) in an effort to normalize the
calcium/phosphate ratio. PTH stimulates osteoblasts to become osteoclasts, which dissolve the bone
reservoir of calcium and release it into the blood. This condition, known as renal osteodystrophy, may
lead to osteomalacia and osteitis fibrosa cystica, which is classically associated with dissecting
osteitis, as described in this question. With respect to parathyroid hormone physiology, the following
apply. The PTH has 5 effects: (1) stimulates osteoclasts; (2) inhibits osteoblasts; (3) promotes
11
intestinal absorption of calcium and phosphate ions; (4) reduces urinary excretion of calcium; (5)
stimulates formation and secretion of calcitrol at the kidneys.
Calcitonin (choice A), which serves to lower serum calcium levels, is produced in small quantities in
chronic renal failure, as serum calcium is already pathologically low.
Excess cortisol (choice B; Cushing’s syndrome) may produce osteoporosis and pathologic fractures,
but there is no relationship between chronic renal failure and cortisol excess. Cushing’s syndrome is
generally secondary to adrenal or pituitary adenomas or primary adrenal hyperfunction.
Erythropoietin (choice C) production is often deficient in chronic renal failure. Excess erythropoietin
activates erythrocyte precursors, but does not activate osteoclasts.
Glucagon (choice D) excess is a very rare entity, occurring in a minority of islet cell tumors, and is not
associated with renal failure. High glucagon produces a transitory skin rash, anemia, and a form of
diabetes mellitus.

How well did you know this?
1
Not at all
2
3
4
5
Perfectly
27
Q
  1. Nonfunctioning of complement component C36 would be expected to interfere with which
    of the following biologic functions?
    A. Decreased appetite
    B. Fever
    C. Increased collagen synthesis by fibroblasts
    D. Increased leukocyte adherence to endothelium
    E. Opsonization to facilitate phagocytosis
A

The correct answer is E. C3b acts as an opsonin, together with IgG, to facilitate phagocytosis. None
of the other functions listed in the answer choices is attributable to C3b. Instead, all of the other
answer choices are functions of the cytokines interleukin-1 (IL-1) and tumor necrosis factor (TNF).
Tumor necrosis factor is produced by macrophages and activated T-cells, whereas many different cell
types (including macrophages) produce IL-1.
Decreased appetite (choice A) is typical during an acute illness (such responses are called acute
phase reactions). Other acute phase reactions include fever (choice B), increased sleep, shock,
leukocytosis, and increased serum acute phase proteins.
Fibroblasts proliferate and increase their synthesis of collagen (choice C), collagenase, protease, and
prostaglandin E in response to IL-1 and TNF.
Effects of IL-1 and TNF on endothelium include increased synthesis of mediators (prostaglandins, IL-
1, IL- 8, platelet-derived growth factor), increased leukocyte adherence (choice D), and increased
procoagulant activity with decreased anticoagulant activity.

How well did you know this?
1
Not at all
2
3
4
5
Perfectly
28
Q
  1. If a creatinine clearance of 100 mL/min changes to 50 mL/min over 1 year, assuming that
    there has been no change in diet, which of the following physiologic changes can be
    expected?
    A. A 2-fold decrease in blood urea nitrogen concentration
    B. A 2-fold decrease in creatinine excretion rate
    C. A 2-fold increase in creatinine excretion rate
    D. A 2-fold increase in creatinine reabsorption
    E. A 2-fold increase in plasma creatinine concentration
A

The correct answer is E. Creatinine clearance is used clinically to estimate glomerular filtration rate
(GFR). The 50% decrease in creatinine clearance in this patient therefore suggests that GFR has
decreased by 50% over the past year. Because creatinine is freely filtered but not reabsorbed (choice
D), the filtration rate and excretion rate of creatinine are equal during steady state conditions. When
GFR decreases, the rate of creatinine excretion will also decrease, causing the rate of creatinine
excretion to decrease to below the rate of creatinine production. The result is an increase in plasma
12
creatinine concentration. When plasma creatinine levels have increased by 2-fold, normal amounts of
creatinine can then again be excreted (compare with choices B and C) because the excretion rate of
creatinine is equal to the product of GFR and plasma creatinine concentration. In summary, when
GFR decreases, the plasma creatinine concentration continues to increase until the rate of creatinine
filtration (and excretion) becomes equal to the rate of creatinine production by the body.
The blood urea nitrogen concentration (choice A) increases when GFR is reduced.

How well did you know this?
1
Not at all
2
3
4
5
Perfectly
29
Q
  1. A cell biologist wants to activate protein kinase C in cultured cells. Stimulation of which of
    the following receptor types would most likely produce the greatest activation?
    A. Alpha 1 adrenergic
    B. Beta 1 adrenergic
    C. Dopamine-2 (D2)
    D. Gamma-aminobutyric acid, type A (GABAA)
    E. Nicotinic cholinergic
A

The correct answer is A. It is a good idea to know the mechanism of action of various receptors.
Alpha-1 adrenergic receptors are coupled to the G protein, Gq. This G protein causes breakdown of
the membrane phospholipid phosphatidylinositol bisphosphate (PIP2), forming the products
diacylglycerol, which stimulates protein kinase C, and inositol triphosphate (IP3), which releases
calcium from the endoplasmic reticulum. It is important to note that cyclic AMP activates protein
kinases by dissociating the complex of regularity and catalytic subunits, which results in activation of
the cellular response.
Beta-1 adrenergic receptors (choice B), like all beta receptors, are coupled to the Gs G protein,
leading to the activation of adenylate cyclase. This increases intracellular concentrations of cAMP by
converting ATP to cAMP. cAMP in turn stimulates protein kinase A, also known as cAMP-dependent
protein kinase.
Dopamine-2 receptors (choice C) are coupled to Gi. Stimulation of these receptors inhibits adenylate
cyclase, thus lowering cAMP levels and protein kinase A activity.
Gamma-aminobutyric acid, type A (GABAA) receptors (choice D) are not G-protein-coupled
receptors, but are instead ligand-gated ion channel receptors. Stimulation of these inhibitory receptors
causes a chloride flux, thus stabilizing the membrane potential of the cell.
Nicotinic cholinergic receptors (choice E) are not G-protein-coupled receptors, but are instead ligandgated
ion channel receptors. Stimulation of these receptors causes sodium influx, leading to excitation
of the cells.

How well did you know this?
1
Not at all
2
3
4
5
Perfectly
30
Q
30. which type of blood vessel is the mean linear velocity of a red blood cell the lowest?
A. Aorta and large arteries
B. Arterioles
C. Capillaries
D. Small arteries
E. Vena cavae and large veins
A

The correct answer is C. The same volume of blood flows through each of the different types of
blood vessels each minute. Because the capillaries have the largest cross-sectional area (averaging
2500-5000 cm2), and the velocity of blood flow is inversely related to cross-sectional area, the mean
linear velocity of a red blood cell is lowest in the capillaries. Under resting conditions, the mean linear
velocity of a red blood cell in the capillaries is 0.3-0.6 mm/sec, whereas the velocity in the aorta
(choice A) is approximately 200 mm/sec. This low velocity of red blood cells in the capillary network
allows plenty of time for oxygen to diffuse to the tissues.
The velocity of blood flow is ranked from highest to lowest as follows: aorta (choice A) > vena cavae
(choice E) > large veins (choice E) > small arteries (choice D) > arterioles (choice B) > capillaries.
13
When the vena cavae are partially collapsed, however, (which occurs often) they have a lower crosssectional
area and a higher velocity of blood flow compared with the aorta.

How well did you know this?
1
Not at all
2
3
4
5
Perfectly
31
Q
31 cells that have a low affinity for MHC class I molecules differentiate in the thymus to
become which type of cell?
A. CD8+ cytotoxic lymphocyte
B. Gamma-delta T cell
C. Natural killer cell
D. T helper 1 cell
E. T helper 2 cell
A

The correct answer is A. CD8 cells are generated after mature T cells interact with certain foreign
antigens. They are responsible for defense against intracellular pathogens, tumor immunity, and
organ graft rejection. CD8+ cytotoxic T lymphocytes are positively selected in the thymus because
they have low affinity for MHC class I molecules. If they had possessed high affinity for the MHC class
I molecules, they would have been eliminated (negative selection) because of the danger of
autoimmune disease. Also, cells with no affinity for MHC class I molecules would be eliminated.
The gamma-delta designation in a T cell (choice B) refers to type of receptor on the cell. Most T-cell
receptors are alpha-beta receptors, but some are of a different isotype termed gamma-delta.
Natural killer cells (choice C) are large granular lymphocytes that are part of the innate immune
response. Natural killer cell function does not depend on MHC class I or class II molecules; it simply
kills tumor cells or virally infected cells.
T helper 1 (choice D) and T helper 2 cells (choice E) would have a low affinity for class II MHC
molecules to survive in the thymus. If they had no affinity or strong affinity for these molecules, they
would have been eliminated.

How well did you know this?
1
Not at all
2
3
4
5
Perfectly
32
Q
32. Maintenance of the corpus luteum during the first trimester of pregnancy is accomplished
principally by the secretion of
A. antidiuretic hormone (ADH)
B. follicle stimulating hormone (FSH)
C. human chorionic gonadotropin (hCG)
D. luteinizing hormone (LH)
E. progesterone
A

The correct answer is C. The corpus luteum secretes estrogens, progesterone, and relaxin. hCG,
secreted by the syncytiotrophoblast lining the placental villi, maintains the corpus luteum during the
first trimester of pregnancy. hCG appears in the maternal blood stream soon after implantation
occurs. This hormone, in addition to maintaining the corpus luteum, also promotes the continued
secretion of progesterone. As a result, the endometrial lining remains functional and menses does not
occur. Antidiuretic hormone (vasopressin; choice A) does not play a significant role during
pregnancy. FSH (choice B) acts on granulosa cells to promote the conversion of androstenedione to
estradiol. LH (choice D) acts on theca cells to promote androstenedione secretion. Progesterone
(choice E) is important for maintaining the pregnancy, however, it does not act to maintain the corpus
luteum.

How well did you know this?
1
Not at all
2
3
4
5
Perfectly
33
Q
33. Metabolism of 100 grams of fat will yield approximately how many kilocalories?
A. 200
B. 400
C. 600
D. 900
E. 1200
A

The correct answer is D. Carbohydrates contain approximately 3.4 kcal of energy and proteins
contain 4 kcal of energy per gram. Fats are the more energy-efficient storage form, containing 9 kcal
of energy per gram. A quantity of 100 grams of fat would therefore yield 900 kcal of energy.

How well did you know this?
1
Not at all
2
3
4
5
Perfectly
34
Q
34. Which of the following hormones of pregnancy is most likely to be significantly decreased
compared with normal in the blood sample of a woman whose fetus has died during the
second trimester?
A. Estriol
B. Human chorionic gonadotropin
C. Human chorionic somatomammotropin
D. Progesterone
E. Prolactin
A

The correct answer is A. Maternal blood levels of estriol, a weak estrogen, depend on a viable fetus.
The fetal adrenal cortex and liver produce the weak androgen, 16-OH dehydroepiandrosterone sulfate
(16-OH DHEA-S) that is carried by the fetal circulation to the placenta, where it is converted to estriol
and secreted into the maternal circulation. Although maternal blood levels of estradiol and estrone
increase by 50-fold during pregnancy, maternal blood levels of estriol increase 1,000-fold. Increasing
maternal blood levels of estriol is the best indicator of fetal well-being. A significant drop in maternal
estriol may indicate fetal jeopardy.
Human chorionic gonadotropin (hCG, choice B) is secreted by syncytiotrophoblast cells and does not
depend on a viable fetus. Human chorionic gonadotropin (hCG) is in the same hormone family as
TSH, FSH, and LH. The maternal blood or urinary level of hCG is used to confirm the presence of
pregnancy. Its function is to “rescue” the corpus luteum and maintain the pregnancy until the placenta
can produce sufficient estrogen and progesterone.
Human chorionic somatomammotropin (hCS; choice C) is secreted by syncytiotrophoblast cells and
does not depend on the presence of a viable fetus. It is also known as human placental lactogen, and
is in the same hormone family as growth hormone and prolactin. Its function in pregnancy is not
completely understood, but it may serve to reduce maternal glucose use and allow for “shunting” of
glucose to the fetus.
Maternal progesterone (choice D) depends on a viable placenta, but does not depend on a fetal
contribution. The precursor for placental secretion of progesterone is maternal (not fetal) cholesterol.
Prolactin (choice E) levels steadily increase during pregnancy. This contributes to the final
development of a mature mammary gland. It is secreted by the anterior pituitary and does not require
a fetal contribution.

How well did you know this?
1
Not at all
2
3
4
5
Perfectly
35
Q
  1. With time, blood stored in a blood bank tends to become relatively depleted of 2,3-
    diphosphoglycerate (2,3-DPG). What effect does this have on the hemoglobin-oxygen
    dissociation curve?
    A. Shifts the curve to the left, so that the hemoglobin has a decreased oxygen affinity
    B. Shifts the curve to the left so that the hemoglobin has an increased oxygen affinity
    C. Shifts the curve to the right, so that the hemoglobin has a decreased oxygen affinity
    D. Shifts the curve to the right, so that the hemoglobin has an increased oxygen affinity
    E. Does not change the dissociation curve
A

The correct answer is B. 2,3-diphosphoglycerate (2,3-DPG) is produced in red cells (RBCs) by a
variation on the glycolytic pathway, and levels diminish when glycolysis by the RBCs slows. The
depletion of 2,3-DPG in stored blood causes the hemoglobin dissociation curve to shift to the left,
leading to an increase in oxygen affinity. This increase is helpful in the picking up of oxygen by
hemoglobin from the lungs, but it can be very problematic in the release of oxygen from the blood to
the tissues. This is not just a theoretical point: considerable effort has been expended in developing
improved solutions for storing packed RBCs and methods for “restoring” older stored cells so that the
2,3-DPG levels are adequate. In practice, in otherwise reasonably healthy patients, older transfused
blood will quickly regenerate 2,3-DPG when placed in the glucose-containing environment of the
serum; however, even transiently decreased 2,3-DPG levels in a severely compromised patient can
be dangerous.

How well did you know this?
1
Not at all
2
3
4
5
Perfectly
36
Q
  1. Under normal conditions, the main drive for respiration is the
    A. arterial PCO2 acting through central chemoreceptors
    B. arterial PCO2 acting through peripheral chemoreceptors
    C. arterial pH acting through central chemoreceptors
    D. arterial pH acting through peripheral chemoreceptors
    E. arterial PO2 acting through central chemoreceptors
A

The correct answer is A. The most important factor in the control of minute-to-minute ventilation is
arterial PCO2, which influences chemoreceptors located near the ventral surface of the medulla. As
arterial PCO2 rises, CO2 diffuses from cerebral blood vessels into the CSF. Carbonic acid is formed
and dissociates into bicarbonate and protons. Protons directly stimulate these central
chemoreceptors, resulting in hyperventilation. Hyperventilating reduces the PCO2 in the arterial blood
and subsequently in the CSF.
Peripheral chemoreceptors located in the carotid and aortic bodies respond to increases in
PCO2(choice B), but are less important than the central chemoreceptors. It is estimated that when a
normal subject hyperventilates in response to inhalation of CO2, less than 20% of the response can
be attributed to the peripheral receptors. However, they respond more quickly than their central
counterparts and are thought to play a role in regulating ventilation after abrupt changes in PCO2.
There are no known central chemoreceptors that respond to either arterial pH (choice C) or arterial
PO2(choice E).
Carotid chemoreceptors (choice D) cause hyperventilation in response to decreases in arterial pH;
however, CO2 acting through central chemoreceptors is the most important regulator of ventilation
under normal conditions.

How well did you know this?
1
Not at all
2
3
4
5
Perfectly
37
Q
37. Which of the following types of nerve fibers carry pain that is aching, burning, sometimes
throbbing, and poorly localized?
A. Group II fibers
B. Group III fibers
C. Type C fibers
D. Substance P
A

The correct answer is C. Slow pain as described in this question is carried by type C fibers. Choice-
A - The dorsal column system consists of type II fibers that carry touch and pressure sensation.
Choice B - Fast pain is carried by group III fibers. It has a rapid onset and offset and is localized. The
anterolateral system, which processes sensations of temperature, pain, and light touch, consists
primarily of group III and group IV fibers. Choice D - Pain is associated with detection and perception
of noxious stimuli (nociception). The neurotransmitters for nociceptors include substance P. Inhibition
of the release of substance P is the basis of pain relief by opioids.

How well did you know this?
1
Not at all
2
3
4
5
Perfectly
38
Q
  1. If a drug is taken that stimulates sympathetic cholinergic neurons, which of the following
    responses is expected?
    A. Bradycardia
    B. Bronchoconstriction
    C. Diaphoresis
    D. Increased gastrointestinal motility
    E. Increased peripheral vascular resistance
A

The correct answer is C. Sympathetic cholinergic neurons are sympathetic postganglionic neurons
that happen to release acetylcholine instead of norepinephrine. The vast majority of sweat glands in
the body are innervated by sympathetic cholinergic neurons (choice C). Bradycardia (choice A),
bronchoconstriction (choice B), and increased gastrointestinal motility (choice D) would all result
from stimulating parasympathetic cholinergic neurons.
16
Increased peripheral vascular resistance (choice E) would result from stimulating sympathetic
adrenergic neurons.

How well did you know this?
1
Not at all
2
3
4
5
Perfectly
39
Q
  1. A 3-year-old child has inhaled a peanut that has lodged in the right mainstem bronchus,
    largely occluding it. The child is cyanotic, with a PO2 of 60 mm Hg. Which of the following
    mechanisms best accounts for the child’s hypoxemia?
    A. Decreased capacity of pulmonary diffusion
    B. Decreased PO2 in inspired air
    C. Hypoventilation of central origin
    D. Hypoventilation of peripheral origin
    E. Inequalities of ventilation and perfusion
A

The correct answer is E. Inequalities of ventilation and perfusion contribute to hypoxia in many
settings. In this case, blood goes to both lungs (perfusion), but air is prevented from entering one of
the lungs (ventilation). Because the right lung is being perfused, but not ventilated, hypoxemia ensues
when the deoxygenated blood from the right lung mixes with oxygenated blood from the left lung. If
the inadequate ventilation of the lung persists long enough, the lung tissue itself can be damaged,
causing a secondary local dilation of arterioles, making the problem even worse. Peanuts are
notorious for producing this type of problem in young children because of their size and shape, which
allow them to lodge in the trachea or main bronchus after aspiration.
Decreased diffusion capacity (choice A) can occur when the blood-gas barrier is thickened (e.g.,
diffuse interstitial fibrosis, sarcoidosis, asbestosis, respiratory distress syndrome), when the surface
area of the blood-gas barrier is reduced (e.g., pneumonectomy, emphysema), or when less
hemoglobin is available to carry oxygen (e.g., anemia, pulmonary embolism).
Decreased PO2 in inspired air (choice B) is seen at high altitudes and when the settings are wrong
during artificial ventilation.
Hypoventilation of central origin (choice C) is seen in morphine and barbiturate overdose.
Hypoventilation of peripheral origin (choice D) is seen in poliomyelitis and chest trauma.

How well did you know this?
1
Not at all
2
3
4
5
Perfectly
40
Q
  1. One hour before a dental procedure a patient receives four 500-mg amoxicillin capsules.
    On arriving at the dental office the dentist notices that the patient has maculopapular rash and
    is wheezing. The effector cell in this type of hypersensitivity is a(n)
    A. eosinophil
    B. mast cell
    C. megakaryocyte
    D. neutrophil
    E. TH1 CD4+ lymphocyte
A

The correct answer is B. This patient is experiencing the early phase of type I hypersensitivity,
characterized by bronchospasm and development of a maculopaplar rash. Other potential symptoms
of this allergic drug reaction include pruritus and watery discharge from the nose, mucus secretion in
the airways, and a wheal-and-flare response with pruritus in the skin. The mechanism of
hypersensitivity involves prior sensitization of a population of TH2 cells that produced cytokines,
including interleukin-4. The interleukin-4 causes B cells to switch their heavy chain class from IgM to
IgE. The IgE molecules then attach to mast cells or basophils. With subsequent antigen (allergen)
challenge, the mast cells degranulate and release mediators, including histamine, which produces the
anaphylactic response.
Eosinophils (choice A) are involved in late-phase type I hypersensitivity reactions and type II
antibody-dependent cell cytotoxicity reactions directed against parasites. Eosinophils enter the area of
the reaction because of the release of eosinophil chemotactic factor (eotaxin) and the beta-chemokine
RANTES released from TH2 CD4+ cells and mast cells. Eosinophils are recruited into the tissues as
17
part of the late-phase reaction of type I hypersensitivity. Their survival in tissue is dependent on IL-3,
IL-5, and granulocyte-monocyte colony stimulating factor (GM-CSF) released from TH2 cells.
Megakaryocytes (choice C) are bone marrow cells that produce platelets. They are not involved in
type I hypersensitivity.
Neutrophils (choice D) are not a cell type that is key to the development of type I hypersensitivity.
TH1 CD 4+ lymphocytes (choice E) are associated with delayed type hypersensitivity reactions such
as contact dermatitis involving exposure to poison ivy or poison oak.

How well did you know this?
1
Not at all
2
3
4
5
Perfectly
41
Q
41. An increase in which of the following is the most likely explanation for the swelling in the
legs seen with congestive heart failure?
A. Interstitial colloid osmotic pressure
B. Lymph flow
C. Plasma colloid osmotic pressure
D. Right atrial pressure
E. Stroke volume
A

The correct answer is D. Congestive heart failure typically develops when the heart becomes
damaged (usually as a result of myocardial infarction) and the cardiac output cannot be maintained at
a normal level. A low cardiac output has profound effects on the kidneys, causing salt and water
retention. If the heart is not damaged too badly, the increase in blood volume caused by the fluid
retention can increase venous return sufficiently to totally compensate for the diminished pumping
capacity of the heart. The increase in venous return causes the right atrial pressure to increase, which
elevates venous pressure throughout the body. This increase in venous pressure can cause
excessive fluid loss from the microcirculation and the development of peripheral edema. The labored
breathing experienced by the patient is indicative of mild pulmonary edema caused by increased
pressure in the pulmonary microvasculature.
The increase in fluid loss from the microcirculation would be expected to literally wash protein
molecules out of the interstitial compartment and thereby decrease the interstitial colloid osmotic
pressure (choice A), and increase the flow of lymph (choice B) from the tissue as a consequence
rather than a cause of the edema.
Increased plasma colloid osmotic pressure (choice C) would tend to decrease the development of
edema. Also, fluid retention by the kidneys tends to dilute the plasma and thus decrease the plasma
colloid osmotic pressure.
Stroke volume (choice E) has no direct role in the formation of peripheral edema.

How well did you know this?
1
Not at all
2
3
4
5
Perfectly
42
Q
42. An increase in which of the following best explains the mechanism by which the cardiac
output increases in severe anemia?
A. Arteriolar diameter
B. Blood viscosity
C. Peripheral vascular resistance
D. Splanchnic blood flow
E. Tissue oxygen tension
A

The correct answer is A. Anemia is present in adults if the hematocrit is less than 41% in men and
37% in women. Poor diet and pathologic bleeding can lead to the development of folic acid deficiency
and iron deficiency anemia, respectively. In severe anemia, diminished transport of oxygen in the
blood leads to hypoxia in the tissues. The hypoxia (compare with choice E) causes small arteries and
arterioles to dilate, which allows greater than normal amounts of blood to return to the heart. In severe
anemia, the viscosity of blood (choice B) may decrease by 50% or more because blood viscosity
depends largely on the concentration of red blood cells. This decrease in viscosity lowers the
18
resistance to blood flow in the peripheral tissues (i.e., decreases peripheral vascular resistance,
choice C) allowing even greater amounts of blood to return to the heart.
Blood is often shunted away from the splanchnic vascular bed (choice D) in anemia, which can cause
gastrointestinal problems.

How well did you know this?
1
Not at all
2
3
4
5
Perfectly
43
Q
43. Which of the following is found in the respiratory zone of the lung?
A. Goblet cells
B. Main bronchi
C. Mucous cells
D. Terminal bronchioles
E. Type I epithelial cells
A

The correct answer is E. There are two zones in the lung: the conducting zone (where there is no
gas exchange) and the respiratory zone (where gas exchange takes place). Of all of the structures
listed, only type I epithelial cells are located in the respiratory zone. Type I epithelial cells are the
primary structural cells of the alveolar wall. Type II epithelial cells, also located in the alveoli, produce
surfactant. Goblet cells (choice A), which are mucus-secreting cells, are present in the conducting
airways. The main bronchi (choice B) are part of the conducting airways. Mucous cells (choice C),
which are mucus-secreting cells, are also present in the conducting airways. Terminal bronchioles
(choice D) are the most distal part of the conducting airways. Respiratory bronchioles, which are just
distal to the terminal bronchioles, are part of the respiratory zone. These two types of bronchioles can
be differentiated from each other by whether they have alveoli budding from their walls. Respiratory
bronchioles have alveoli, terminal bronchioles do not.

How well did you know this?
1
Not at all
2
3
4
5
Perfectly
44
Q
44. Laboratory studies of an automobile accident victim show a significant elevation of
creatine kinase (CK). Which of the following tissues is most likely to be the source of the
enzyme?
A. Bone
B. Brain
C. Heart
D. Kidney
E. Skeletal muscle
A

The correct answer is E. Creatine kinase occurs in several different forms, or isoenzymes, in the
body. Most tissues contain a mix of creatine kinase isoenzymes, but one species often predominates.
The MB isoenzyme of creatine kinase is associated with heart damage; the MM isoenzyme is
associated with muscle damage; and the BB isoenzyme is associated with brain damage. Surgery
and general trauma are commonly implicated as causes of sufficient skeletal muscle trauma to
markedly elevate creatine kinase by way of the CK-MM (skeletal muscle isoenzyme) isoenzyme.
Bone (choice A) is not a significant source of CK. Brain (choice B) can occasionally be a source of
creatine kinase (BB isoenzyme), but would not usually be the major source following an automobile
accident. Heart damage (choice C), as in myocardial infarction, can elevate creatine kinase, but
would not usually be the major source in an automobile accident. If the dilemma arises as to whether
an automobile accident was possibly secondary to a myocardial infarction, the ratio of CK-MB to total
CK may be helpful in demonstrating a cardiac contribution (CK-MB is characteristic of heart muscle).
Kidney (choice D) is not a major source of creatine kinase.

How well did you know this?
1
Not at all
2
3
4
5
Perfectly
45
Q
45. Which of the following binds to von Willebrand factor at the platelet membrane?
A. Adenosine diphosphate (ADP)
B. Calcium
C. Glycoprotein GPIb
D. Platelet factor 3 (PF3)
E. Prostacyclin
A

The correct answer is C. Glycoprotein GPIb on the platelet membrane binds von Willebrand factor, a
plasma protein that circulates in a complex with factor VIII. ADP (choice A) is a powerful inducer of
platelet aggregation and strengthens the platelet plug by the addition of more activated platelets.
Calcium (choice B) is essential for increasing the degree of platelet aggregation and strengthening
the platelet plug. It is also a necessary cofactor in the coagulation cascade (required for the
conversion of factors IX to IXa, X to Xa, V to Va, and prothrombin [factor II] to thrombin [factor IIa]).
Platelet factor 3 (PF3; choice D) is involved in platelet plug formation. Prostacyclin (choice E) is
synthesized by blood vessel endothelial cells and inhibits platelet aggregation.

How well did you know this?
1
Not at all
2
3
4
5
Perfectly
46
Q
  1. Which of the following statements about Troponin C is true? Troponin C
    A. inhibits the interaction of actin and myosin
    B. attaches the troponin complex to tropomyosin
    C. permits the interaction of actin and myosin
    D. is located at the junction of A band and I band
A

The correct answer is C. Troponin C is the calcium binding protein that when bound to calcium
permits the interaction of actin and myosin. Cross bridges continue as long as calcium is bound to
troponin C. Choice A - Troponin I inhibits the interaction of actin and myosin. Choice B - Troponin T
attaches the troponin complex to tropomyosin. Choice D - T tubules are the extensive tubular network
that carry the depolarization from sarcolemmal membrane to the cell interior. These are located at the
junction of A band and I band.

How well did you know this?
1
Not at all
2
3
4
5
Perfectly
47
Q
47 .Which of the following can be determined by calculation of the clearance of
paraaminohippuric acid (PAH)?
A. Extracellular fluid (ECF) volume
B. Effective renal plasma flow (ERPF)
C. Glomerular filtration rate (GFR)
D. Plasma volume
E. Total body water (TBW)
A

The correct answer is B. At less than saturating concentrations, PAH is completely secreted into the
proximal tubule and excreted into the urine. The volume of plasma cleared of PAH is therefore
approximately equal to the volume of plasma flowing through the peritubular capillaries, also called
the effective renal plasma flow, or ERPF. The ECF volume (choice A) can be calculated by
measuring the volume of distribution of solutes that move freely across capillary walls but cannot
permeate cell membranes (e.g., inulin and mannitol). GFR (choice C) is best calculated using a
substance that is freely filtered at the glomerulus, not reabsorbed, and only minimally secreted into
the urine. Creatinine fits the bill and is used clinically to measure the GFR (inulin also works and is
used experimentally). Although the creatinine excretion exceeds filtration by 10-20% (because of the
secretion), creatinine clearance is still a good approximation for GFR because the error caused by
secretion is balanced by an overestimation of plasma creatinine inherent in the measurement
technique. The plasma volume (choice D) can be measured by measuring the volume of distribution
of radioactively labeled serum albumin or of Evans blue dye (binds to albumin). Total body water
(TBW; choice E) can be measured by measuring the volume of distribution of tritium, deuterium, or
antipyrine.

How well did you know this?
1
Not at all
2
3
4
5
Perfectly
48
Q
  1. The clearance of several substances was measured at a constant glomerular filtration rate
    and constant urine flow rate, but at increasing plasma concentrations of the substance. Under
    these conditions, clearance will increase at high plasma concentrations for which of the
    following substances?
    A. Creatinine
    B. Mannitol
    C. Penicillin
    D. Phosphate
    E. Urea
A

The correct answer is D. Clearance of a substance will change with increasing plasma
concentration if that substance is secreted or reabsorbed by a facilitated mechanism. As the
20
concentration of the substance increases, the transporter becomes saturated, and its contribution to
excretion changes, changing the clearance. If the substance is reabsorbed by a facilitated
mechanism, clearance will eventually increase with increasing plasma concentrations. Approximately
80% of filtered phosphate is reabsorbed in the proximal tubule by a sodium-phosphate cotransporter,
which is a facilitated mechanism. The phosphate buffer system consists of an anion, H2PO4, that is a
weak acid. The basic operation of the phosphate buffer system resembles that of the carbonic acidbicarbonate
buffer system.

How well did you know this?
1
Not at all
2
3
4
5
Perfectly
49
Q
49. Gastrin stimulates the secretion of HCl. Which of the following cell types located at the
fundus of the stomach produces HCl?
A. Parietal cells
B. Chief cells
C. G cells
D. Mucous cells
A

The correct answer is A. Parietal cells located in the body of the stomach secrete HCl and intrinsic
factor. Stimuli for secretion are gastrin, histamine, and Ach. Parietal cells secrete HCl into the lumen
of the stomach and concurrently absorb bicarbonate into the blood stream. Choice B - Chief cells,
located in fundus, secrete pepsinogen under vagal stimulation. Choice C - Vagal stimulation secretes
gastrin from the G cells of the antrum of the stomach. Choice D - Mucous cells, located in the antrum
of the stomach, secrete mucus and pepsinogen under vagal stimulation.

How well did you know this?
1
Not at all
2
3
4
5
Perfectly
50
Q
  1. Adrenocorticotropic hormone (ACTH) promotes the production of cortisol by stimulating
    which of the following reactions?
    A. 11-Deoxycortisone to cortisol
    B. 17-Hydroxyprogesterone to 11-deoxycortisol
    C. Cholesterol to pregnenolone
    D. Pregnenolone to progesterone
    E. Progesterone to 17-hydroxyprogesterone
A

The correct answer is C. All of the choices listed are reactions that occur in the synthetic pathway
from cholesterol to cortisol. ACTH stimulates the first reaction in the pathway: cholesterol to
pregnenolone. This reaction is catalyzed by the enzyme cholesterol desmolase.
The next step in the pathway is pregnenolone to progesterone (choice D); progesterone is then
converted to 17-hydroxyprogesterone (choice E); 17-hydroxyprogesterone is converted to 11-
deoxycortisol (choice B) by the enzyme 21 beta-hydroxylase; and the 11-deoxycortisol is then
converted to cortisol (choice A).

How well did you know this?
1
Not at all
2
3
4
5
Perfectly
51
Q
  1. Laboratory studies for an alcoholic patient indicate that serum magnesium is depressed.
    Which of the following findings would be most consistent with this information?
    A. Decreased serum calcium
    B. Decreased serum phosphate
    C. Increased bone density
    D. Increased plasma parathyroid hormone concentration
A

The correct answer is A. Malnutrition associated with chronic alcoholism can lead to a severe
magnesium deficiency. The effect of low serum magnesium on parathyroid hormone secretion (PTH)
depends on severity and duration. An acute decrease in serum magnesium will increase PTH
secretion, whereas a prolonged severe deficiency results in decreased PTH secretion. There is also
evidence that the action of PTH is decreased with chronic magnesium deficiency. Hence, this patient
is suffering from “functional” hypoparathyroidism, causing low serum calcium (choice A) that can
produce weakness, tremors, muscle fasciculations, and seizures. In patients with magnesium
deficiency, magnesium administration will produce a prompt increase in plasma PTH with subsequent
restoration of serum calcium concentration to normal.
21
With functional hypoparathyroidism bone density would be decreased (not increased, choice C). The
combination of decreased PTH secretion (not increased, choice D) and decreased effectiveness of
PTH produce hypocalcemia and hyperphosphatemia (not hypophosphatemia, choice B).
Phosphorous is needed for development of high-energy compounds, nucleic acids, and bone matrix.
Respiratory rate 15; blood pressure 120/80 mm Hg; cardiac output 5 L/min; pulse 50

How well did you know this?
1
Not at all
2
3
4
5
Perfectly
52
Q
52. Basal measurements are shown above. What is the stroke volume during resting
conditions (in mL/min)?
A. 50
B. 75
C. 100
D. 125
E. 150
A

The correct answer is C. The cardiac output (CO) is equal to the volume of blood ejected from the
heart during each systole (i.e., the stroke volume; SV) multiplied by the number of times the heart
beats each minute (heart rate; HR). In other words, CO = SV x HR. Therefore, SV = CO/HR, and
because CO = 5,000 mL/min and HR = 50 beats/min, SV = 5,000/50 = 100 mL/beat.
In the examination of a single cardiac cycle, stroke volume is the most important factor to consider. In
the consideration of cardiac function over time, clinicians are concerned with cardiac output (the
volume of blood ejected in 1 minute). Physiologically, cardiac output is precisely regulated so that
peripheral tissues receive an adequate circulatory supply under a variety of conditions; for example,
stroke volume can almost triple when neccessary.

How well did you know this?
1
Not at all
2
3
4
5
Perfectly
53
Q
  1. Which of the following would most likely be an associated finding during fasting for a
    patient with an insulin-secreting tumor that is localized to the tail of the pancreas?
    A. Glycosylated hemoglobin level is increased
    B. Plasma concentration of C peptide is decreased
    C. Plasma concentration of glucagon is decreased
    D. Plasma concentration of glucose is increased
    E. Plasma concentration of proinsulin is increased
A

The correct answer is E. Hypersecretion of insulin by a pancreatic cell tumor is a major cause of
fasting hypoglycemia (plasma glucose is not increased, choice D). Symptoms are related to
neuroglycopenia and could include recurrent central nervous system dysfunction during fasting or
exercise. Although proinsulin only makes up approximately 20% of plasma immunoreactive insulin in
normal individuals, in patients with an insulinoma it contributes 30-90% of the immunoreactive insulin.
The increased secretion of insulin by the tumor will also lead to an increase in C peptide secretion
(not decreased, choice B) since ß cells secrete insulin and C peptide on a one-to-one molar ratio.
Given the prolonged hypoglycemia, the amount of glycosylated hemoglobin may also be decreased,
although this is not a universal finding. Certainly an increase in glycosylated hemoglobin would not be
expected (choice A). Glucagon secretion is increased by hypoglycemia and its plasma level in a
patient with an insulinoma would be expected to be increased compared with normal (not decreased,
choice C).

How well did you know this?
1
Not at all
2
3
4
5
Perfectly
54
Q
  1. Physical examination of a woman with advanced, metastatic lung cancer demonstrates
    hyperpigmentation of skin, even in areas protected from the sun. ACTH levels are found to be
    elevated. Tumor inducement of which endocrine organ is most likely?
    A. Adrenal gland
    B. Endocrine pancreas
    C. Ovaries
    D. Pituitary gland
    E. Thyroid gland
A

The correct answer is A. This is Addison disease, in which severe adrenal disease produces
adrenocortical insufficiency. Causes include autoimmune destruction, congenital adrenal hyperplasia,
hemorrhagic necrosis, and replacement of the glands by either tumor (usually metastatic) or
granulomatous disease (usually tuberculosis). The symptoms can be subtle and nonspecific (such as
22
those listed). Skin hyperpigmentation is a specific clue that may be present on physical examination,
suggesting excess pituitary ACTH secretion. Most patients have symptoms (fatigue, gastrointestinal
distress) related principally to glucocorticoid deficiency. In some cases, however, mineralocorticoid
replacement may also be needed for symptoms of salt wasting with lower circulating volume.
Except in the case of primary pancreatic cancer, complete tumor replacement of the endocrine
pancreas (choice B) would be uncommon. In any event, pancreatic involvement would be associated
with diabetes mellitus.
Involvement of the ovaries (choice C) by metastatic tumor (classically gastric adenocarcinoma) would
produce failure of menstruation.
Involvement of the pituitary gland (choice D) could produce Addisonian symptoms, but the pigmented
skin suggests a primary adrenal problem rather than pituitary involvement.
Replacement of the thyroid gland (choice E) by tumor would cause hypothyroidism with lesser
degrees of fatigue and cold intolerance; this is an unusual cause of hypothyroidism.

How well did you know this?
1
Not at all
2
3
4
5
Perfectly
55
Q
  1. The resultant decrease in blood flow causes by partial obstruction of the renal artery by an
    atherosclerotic plaque increases release of an enzyme from which of the following structures?
    A. Afferent arterioles
    B. Arcuate arteries
    C. Juxtaglomerular cells
    D. Kupffer cells
    E. Proximal convoluted tubule
A

The correct answer is C. The juxtaglomerular cells are in the wall of the afferent arteriole, close to
the glomerulus. Together the macula densa and juxtaglomerular cells form the juxtaglomerular
apparatus. In response to decreased blood pressure, they secrete renin, an enzyme that converts
angiotensinogen to angiotensin I. Angiotensin converting enzyme, found in the lungs, converts
angiotensin I to angiotensin II. Angiotensin II increases peripheral vascular resistance directly and
stimulates aldosterone secretion, resulting in increased reabsorption of sodium and water in the distal
convoluted tubules.
The afferent arteriole (choice A) carries blood from the interlobular arteries to the glomerulus.
Filtration of blood occurs in the glomerulus, with the filtrate entering Bowman’s capsule.
The arcuate arteries (choice B) are branches of the interlobar arteries of the kidney. The arcuate
arteries lie in the corticomedullary junction of the kidney and give rise to interlobular arteries, which
enter the cortex of the kidney and supply the glomeruli.
Kupffer cells (choice D) are found in the liver, along the sinusoids. They are phagocytic cells that are
part of the reticuloendothelial system.
The proximal convoluted tubule (choice E) is directly continuous with Bowman’s capsule. Most of the
resorption of the glomerular filtrate occurs in this part of the nephron.

How well did you know this?
1
Not at all
2
3
4
5
Perfectly
56
Q
56. In a person weighing 75 kg, the volume of total body water is 45 L. What is the interstitial
volume and plasma volume?
A. 4 L, 12 L
B. 12 L, 4 L
C. 8 L, 8 L
D. 5 L, 10 L
A

The correct answer is B. Interstitial volume = 12 L and plasma volume = 4 L. The calculation is
based on the following estimates: of total body weight, approximately 60% is water. One third of this
water is extracellular, and two thirds is intracellular. Of the one third that is extracellular, approximately
one quarter is plasma volume, and the other three quarters is interstitial volume. Using the figures in
the problem:

How well did you know this?
1
Not at all
2
3
4
5
Perfectly
57
Q
57. When a 10-day-old infant jumps and jerks his arms to his chest in response to a noice, the
infant is exhibiting the
A. Babinski reflex
B. deep tendon reflex
C. Moro reflex
D. palmar reflex
E. tonic neck reflex
A

The correct answer is C. The identification of reflexes is an important part of human physiology. The
Moro reflex can be elicited in the infant by any startling event; it consists of extension and abduction of
the arms, followed by flexion and adduction of the arms. This is a normal reflex that appears between
the ages of 25 and 36 weeks of gestation, and will normally disappear between 3-6 months of age.
The Babinski reflex (choice A) is elicited when the lateral surface of the sole of the foot is stroked
resulting in the great toe going up and the other toes fanning. It normally disappears at 1 year of age.
Deep tendon reflexes (choice B) can be elicited by tapping a tendon with a reflex hammer, which
stretches the tendon, producing contraction in the corresponding muscle. These are present
throughout life.
The palmar grasp reflex (choice D) is characterized by the infant’s hand closing over an object that is
placed in the palm of the hand. This reflex normally disappears at 2 months of age.
The tonic neck reflex (choice E) consists of extension of the ipsilateral leg and flexion of the
contralateral arm and leg when the head is turned. This reflex normally disappears between 7-8
months of life.

How well did you know this?
1
Not at all
2
3
4
5
Perfectly
58
Q
58. Which of the following is the primary opsonin in the complement system?
A. C1q
B. C3b
C. C5
D. C5a
E. Factor B
A

The correct answer is B. Complement consists of 11 plasma proteins that interact in a chain reaction
following exposure to activated antibodies or surfaces of certain pathogens. Complement promotes
cell lysis, phagocytosis, and often defense mechanisms. C3b is the most critical molecule in both the
classical and alternative complement pathways. C3, the most abundant protein of all the complement
proteins, is cleaved into C3a and C3b. C3b attaches to bacterial surfaces for opsonization by
phagocytes. C3a binds to mast cells and basophils, activating them and producing histamine release.
C1q (choice A) is a complement component in the classical pathway. It binds to the constant heavy
domain of an IgG molecule that has reacted with the bacterial surface epitope. C1q is not involved in
opsonization as it simply functions as an enzyme in the early complement cascade.

How well did you know this?
1
Not at all
2
3
4
5
Perfectly
59
Q
  1. Constriction of which of the following types of blood vessels is most important for
    minimizing the decrease in mean systemic filling pressure caused by loss of 1 liter of blood?
    A. Aorta and large arteries
    B. Arterioles
    C. Capillaries
    D. Small arteries
    E. Venules and veins
A

The correct answer is E. The venous system serves as an important blood reservoir for the
circulation. When blood is lost from the body and blood pressure begins to decrease, pressure
24
reflexes are elicited that send sympathetic nerve signals to venules and veins causing them to
constrict. By “tightening” the circulation and taking up much of the “slack” caused by the blood loss,
nearly normal function can usually be restored with up to a 20% loss of blood volume.
Constriction of the arterial system has relatively little effect on mean systemic filling pressure because
the arterial system (choices A, B, and D) contains a relatively small volume of blood.
Capillaries do not constrict because they do not contain smooth muscle cells in their walls (choice C).

How well did you know this?
1
Not at all
2
3
4
5
Perfectly
60
Q
  1. Which of the following regulates the frequency of gastrointestinal slow waves?
    A. Circadian rhythms
    B. Dietery intake of fiber
    C. Intrinsic pacemaker regions within the smooth muscle
    D. Local levels of cholecystokinin secretion
    E. Systemic levels of catecholamines and corticosteroids
A

The correct answer is C. Slow waves are oscillating membrane potentials that are generated locally
within the gastrointestinal smooth muscle. In each region of the gastrointestinal tract, intrinsic
pacemaker cells within the smooth muscle independently generate slow waves. Slow-wave
frequencies are relatively constant and are characteristic of each region of the gut.
There is no evidence that either circadian rhythms (choice A) or dietary fiber intake levels (choice B)
have any effect on gastrointestinal slow wave frequencies.
Cholecystokinin (choice D), corticosteroids, and sympathetic stimulation (choice E) may modify the
frequency of action potentials and contractions by affecting the amplitude, rather than the frequency,
of the slow waves.

How well did you know this?
1
Not at all
2
3
4
5
Perfectly
61
Q
  1. A 72-year-old woman with mild heart failure is treated overzealously with a thiazide
    diuretic. A few days later, the woman complains of muscle weakness, and laboratory tests
    demonstrate hypokalemia. Which of the following is most likely increased in this woman?
    A. Arterial H+ concentration
    B. Plasma aldosterone
    C. Plasma sodium
    D. Potassium retention
    E. Sodium retention
A

The correct answer is B. Aldosterone is a minerlocorticoid produced by the zona glomerulosa or the
adrenal cortex. It stimulates sodium and water conservation at the kidneys. This hormone is secreted
in response to the presence of angiotensin II. Diuretics lead to aldosterone excess and hypokalemia
by a variety of mechanisms. (1) Diuretic induces volume depletion stimulated the formation of
angiotensin II, which, in turn, causes a secondary increase in plasma aldosterone concentration. This
increase in plasma aldosterone stimulates potassium excretion, contributing to the hypokalemia. (2)
The saline diuresis increases sodium delivery to the collecting tubule. The increased availability of
sodium along with the elevated plasma aldosterone augments sodium reabsorption in the collecting
tubule, thereby raising luminal negativity. This high luminal negativity in the collecting tubule promotes
secretion of cations, especially hydrogen ions, which raises bicarbonate reabsorption. (3) The saline
diuresis causes rapid fluid flow in the distal tubule, which, in turn, keeps luminal potassium
concentration low by carrying it away and thus preventing the assimilation of any potassium that
enters the lumen. This low luminal concentration of potassium creates a steep concentration gradient
for additional potassium losses in the urine. The treatment of edema with thiazide or loop diuretics is
the most common cause of metabolic alkalosis. Arterial pH is increased and arterial H+ concentration
(choice A) is decreased with metabolic alkalosis.
Sodium depletion tends to decrease plasma sodium (choice C) levels, although the effect is usually
small. Overuse of the thiazide diuretic has caused depletion of sodium and potassium by the kidneys,
not retention of sodium and potassium (choices D and Choice E).

How well did you know this?
1
Not at all
2
3
4
5
Perfectly
62
Q
62. Increasing urine flow rate from 1 mL/min to 10 mL/min will significantly increase the
clearance of
A. creatinine
B. inulin
C. penicillin
D. phosphate
E. urea
A

The correct answer is E. Urine flow rate is controlled primarily by antidiuretic hormone (ADH), which
regulates the amount of pure water retained in the urine. ADH also controls the reabsorption of urea in
the papillary collecting duct. High urine flow rates indicate low ADH, which increases urea clearance.
In contrast, low urine flow rates indicate high ADH, which results in a greater reabsorption of urea and
a lower urea clearance.
The concentration (and osmolarity) of all other solutes (choices A, B, C, and D) varies inversely with
urine flow rate, resulting in no change in clearance. Recall that clearance = (urine concentration x
urine flow rate)/plasma concentration.

How well did you know this?
1
Not at all
2
3
4
5
Perfectly
63
Q
  1. A 3-year-old boy with chronic granulomatous disease (CGD) has a culture of a purulent
    discharge from an abscess that shows Staphylococcus aureus. Immunoglobulin and
    complement levels are normal. The boy received all of the standard immunizations without any
    adverse effects. His immune deficiency most likely involves which of the following?
    A. B cells
    B. Chemotaxis
    C. IgG subclass 2
    D. Phagocytic cells
    E. T cells
A

The correct answer is D. This patient has chronic granulomatous disease (CGD), which is
associated with defective phagocytes. CGD is inherited as an X-linked or autosomal recessive trait
and consists of a group of heterogenous disorders of oxidative metabolism affecting the normal
respiratory burst, consisting of increased oxygen consumption, leading to the generation of
intracellular hydrogen peroxide and superoxide radicals by phagocytic cells. The hydrogen peroxide
and superoxide radicals are required for the killing of ingested intracellular organisms. The reaction is
catalyzed by an NADPH oxidase that seems to be defective in the phagocytes of patients with CGD.
In CGD patients, the engulfment process by the phagocytic cells is normal; however, the pathogenic
organism is not killed, but persists within the cell. The patients suffer from infections with organisms
that are normally considered of low virulence (e.g., Staphylococcus aureus, Aspergillus, Candida,
Escherichia coli, and Serratia marcescens). The nitroblue tetrazolium (NBT) test is used to screen for
CGD. B cells (choice A) are responsible for antibody-mediated immunity, and the immunoglobulin
levels in this patient were normal. Chemotaxis (choice B) is important in the migration of the
phagocytic cell toward the site of infection, not in intracellular killing. IgG subclass 2 (choice C) is the
most important immunoglobulin in the protection against encapsulated organisms. T cells (choice E)
are important in the host response to viruses, fungi, and intracellular bacterial organisms. The patient
was immunized normally (including the live, attenuated MMR vaccine), making T-cell dysfunction
unlikely.

How well did you know this?
1
Not at all
2
3
4
5
Perfectly
64
Q
  1. Which of the following indices would be expected to be higher in an individual living at sea
    level versus an individual living in the mountains?
    A. Diameter of pulmonary vessels
    B. Erythropoietin production
    C. Mitochondrial density in a muscle biopsy
    D. Renal bicarbonate (HCO3-) excretion
    E. Respiratory rate
A

The correct answer is A. Several physiologic changes occur in a person living at high altitude. The
diminished barometric pressure at high altitude causes alveolar hypoxia and arterial hypoxia.
Pulmonary vasoconstriction occurs in response to alveolar hypoxia; therefore, the diameter of the
pulmonary vessels would be greater in the brother living at sea level. All the other choices describe
physiologic processes that would be enhanced by living at high altitude.
Increased erythropoietin production (choice B) caused by arterial hypoxia leads to increases in
hematocrit and increased oxygen carrying capacity in people living at high altitude.
Mitochondrial density increases (choice C) in people chronically exposed to the hypoxemia caused
by living at high altitude.
At high altitudes, the ventilation rate increases, causing a respiratory alkalosis. The kidney then
compensates by increasing the excretion of HCO3- (choice D).
Increasing the rate of respiration (choice E) is a very useful adaptation to the hypoxic conditions of
high altitude. The primary stimulus is the hypoxic stimulation of peripheral chemoreceptors.

How well did you know this?
1
Not at all
2
3
4
5
Perfectly
65
Q
  1. Which of the following substances will be elevated in the plasma of a patient with
    nephrotic syndrome caused by membranous glomerulonephritis?
    A. Albumin
    B. Ammonia
    C. Cholesterol
    D. Glucose
    E. Potassium
A

The correct answer is C. Nephrotic syndrome is characterized by urinary protein excretion,
hypoalbuminemia and peripheral edema. The nephrotic syndrome describes a group of laboratory
findings associated with glomerular diseases that share the common characteristic of “leaky
glomeruli.” Large biochemicals, normally unable to cross out of glomerular capillaries into Bowman
space, are lost into the urine. Serum proteins are lost in large quantities and can be detected as both
hypoproteinemia and massive proteinuria. Albumin (choice A), a relatively small plasma protein (MW
66,000) will be lost very readily, leading to hypoalbuminemia. Serum concentrations of small
compounds such as potassium (choice E) and glucose (choice D), which are highly permeable in the
normal glomerulus, are unaffected by glomerulonephritis producing the nephrotic syndrome. Similarly,
blood urea nitrogen is unaffected, and serum ammonia levels (choice B) are unchanged. The final
component of the nephrotic syndrome (besides hypoproteinemia, hypoalbuminemia and massive
proteinuria) is hyperlipidemia. It is apparently a function of both increased hepatic fat synthesis and
decreased fat catabolism. Increased cholesterol (choice C), triglycerides, and lipoproteins are found
in serum in membranous glomerulonephritis, and these lipids also leak into the urine, producing
lipiduria.

66
Q
  1. An endometrial biopsy demonstrates a thick endometrium with long, coiled glands lined
    by a columnar epithelium with prominent cytoplasmic vacuoles adjacent to the gland lumen.
    Earlier in the menstrual cycle, the glands were much smaller and were lined with cells that did
    not have vacuoles. Which of the following hormones is primarily responsible for inducing this
    change in appearance?
    A. Aldosterone
    B. Cortisol
    C. Estrogen
    D. Progesterone
    E. Thyroxine
A

The correct answer is D. The endometrial phase with small glands is the proliferative phase; the
one with large glands with secretory cells is the secretory phase. Estrogen (choice C) is necessary
for both phases, but it is the addition of progesterone (choice D), secreted by the corpus luteum after
the Graafian follicle ruptures, that triggers the switch from proliferative to secretory endometrium.
27
Progesterone prepares the uterus for implantation and prepares mammary glands for secreting
functions. Progesterone is stimulated by leutinizing hormone from the anterior pituitary.
Glucocorticoids (choice B) and the mineralocorticoid aldosterone (choice A) are secreted by the
adrenal glands. They do not produce the endometrial changes described. Glucocorticoids cause a
release of amino acids from skeletal muscles, lipids from adipose tissue and promote liver glycogen
and glucose formation. They also have anti-inflammatory effects. Mineralocorticoids increase renal
absorption of sodium ions and water and accelerate urinary loss of potassium ions. Thyroxine (choice
E) is secreted by the thyroid gland, and is unrelated to the observed morphologic changes in the
endometrium. It is responsible for metabolic regulation.

67
Q
67. In which segment of the systemic circulation does the greatest decrease in blood pressure
occur?
A. Aorta and large arteries
B. Arterioles
C. Capillaries
D. Small arteries
E. Venules and small veins
A

The correct answer is B. As blood flows through the systemic circulation the mean pressure of the
blood decreases from approximately 100 mm Hg in the aorta to approximately 0 mm Hg in the right
atrium. The mean blood pressure is approximately the same in all portions of the aorta (choice A)
and it only decreases by a few mm Hg in the large arteries (choice A). The blood pressure decreases
by 10-20 mm Hg in the small arteries (choice D) so that blood entering the arterioles has a pressure
averaging 80-90 mm Hg. By the time the blood has reached the ends of the arterioles (choice B) the
pressure has decreased to approximately 35 mm Hg. The pressure decreases another 25 mm Hg as
it flows through the capillary network (choice C), so that blood entering the venules has a pressure of
approximately 10 mm Hg. The blood pressure decreases by approximately 10 mm Hg as it flows
along the venous system (choice E) to the right atrium. This decrease in blood pressure (expressed
as percent of total peripheral resistance)(from 100 to 0 mm Hg) through the circulatory tree is
summarized in the following table. Type of blood vessel: decrease in blood pressure (mm Hg)
(percent of total peripheral resistance) through the circulatory tree. Aorta and large arteries: less than
1; Small arteries: 10-20 ; Arterioles: 50 ; Capillaries: 25 ; Venules and small veins: 9 ; Vena cava: less
than 1

68
Q
  1. Which of the following adjustments have probably occurred in the cardiac cycle if the
    pulse is greater than 200/min and the blood pressure is 75/40 mm Hg?
    A. Diastolic time has decreased and systolic time had increased
    B. Diastolic time has decreased but systolic time had decreased more
    C. Systolic time has decreased and diastolic time has increased
    D. Systolic time has decreased but diastolic time has decreased more
    E. Systolic time has decreased but diastolic time has not changed
A

The correct answer is D. Under normal conditions one third of the cardiac cycle is spent in systole
and two thirds is spent in diastole. As pulse increases dramatically, the time spent in diastole
decreases precipitously but the time spent in systole decreases only slightly. A large increase in pulse
must produce a decrease in diastole and systole (compare with choice A). The major change with
increased pulse is in diastole, not systole (compare with choice B). Pulse cannot increase if diastolic
time increases (choice C). An increase in pulse must be accompanied by a decrease in diastolic time
(compare with choice E).

69
Q
  1. Following a serious accident, a patient’s rib cage and abdominal muscles become
    completely paralyzed. He is still able to breathe, however, because his diaphragm continues to
    contract. At which level might his spinal cord injury have occurred?
    A. C2
    B. C7
    C. L3
    D. T5
    E. T12
A

The correct answer is B. Trauma to the lower cervical cord (at C7, for example) can cause the
pattern described in the question, because the lesion is below the origin of the phrenic nerve (C3-C5
nerve roots), but above the origin of the nerves innervating the muscles of the rib cage and abdomen.
Trauma high in the neck at C2 (choice A), above the origin of the phrenic nerve, would cause
diaphragmatic paralysis also. A lesion at L3 (choice C)or T12(choice E) would spare all of the
accessory respiratory muscles and the diaphragm. A lesion at T5 (choice D) would spare part of the
accessory muscles of the rib cage and the diaphragm.

70
Q
  1. Which of the following events occurs first in the differentiation sequence of human B cells
    in the bone marrow?
    A. Cytoplasmic mu chains present in the B cell
    B. Immunoglobulin heavy chain rearrangement
    C. Immunoglobulin light chain rearrangement
    D. Surface IgD and IgM present on the B cell
    E. Surface IgM present on the B cell
A

The correct answer is B. The first event that occurs in the pre-B cell (progenitor) is gene
rearrangement of the heavy chain. The D gene and J gene recombination event occurs first, followed
by V recombination with the D-J region.
The cytoplasmic mu chains (choice A) are the result of immunoglobulin heavy chain rearrangement,
the production of the messenger RNA from this rearrangement, and the ribosomal synthesis of the
protein mu chain. This is the second event that occurs in the B-cell maturational sequence.
Light chain rearrangement (choice C) occurs when recombination events occur with the V gene and J
gene from the light chain germ line. After this recombination has occurred, and the messenger RNA
for this germ line has produced the light chain protein, the light chains and heavy chains form.
The complete IgM molecule and IgD molecules (choice D) are present on the surface of only the
mature B cells. This event is the last event to occur during the differentiation and development of B
cells in the bone marrow.
The complete IgM molecule (choice E) is present on the surface of the immature B cell. This event is
one of the last events to occur in the differentiation and development of B cells.

71
Q
  1. A patient experiences episodic abdominal pain, especially after the ingestion of a fatty
    meal. The action of which of the following hormones is most likely to be responsible for the
    postprandial intensification of the symptoms?
    A. Cholecystokinin (CCK)
    B. Gastrin
    C. Pepsin
    D. Secretin
    E. Somatostatin
A

The correct answer is A. This symptomatology is consistent with gallstones (cholelithiasis). As would
be expected, contraction of the gallbladder following a fatty meal often exacerbates the pain caused
by gallstones. Cholecystokinin (CCK) is the hormone responsible for stimulation of gallbladder
contraction; the release of CCK is stimulated by dietary fat. It is produced in I cells of the duodenum
and jejunum. In addition to gallbladder contraction, CCK also stimulates pancreatic enzyme secretion
and decreases the rate of gastric emptying.
Gastrin (choice B) is produced by the G cells of the antrum and duodenum. Gastrin stimulates the
secretion of HCl from the parietal cells and pepsinogen from the chief cells of the stomach. Gastrin
secretion is stimulated by gastric distention, digestive products (e.g., amino acids), and vagal
discharge. Pepsin (choice C) is a protease produced by the chief cells of the stomach (as
pepsinogen). It is involved in the digestion of proteins. Pepsinogen release is stimulated by vagal
29
stimulation, gastrin, local acid production, secretin, CCK, and histamine. Secretin (choice D) is
produced by the S cells of the duodenum. It is secreted primarily in response to acidification of the
duodenal mucosa. Secretin stimulates the secretion of bicarbonate-containing fluid from the pancreas
and biliary ducts. This neutralization allows pancreatic enzymes to function. Secretin also inhibits
gastric acid production and gastric emptying.
Somatostatin (choice E) is produced by the D cells of the pancreatic islets and in the gastric and
intestinal mucosa. Somatostatin is an inhibitory hormone; it inhibits most gastrointestinal hormones,
gallbladder contraction, gastric acid and pepsinogen secretion, pancreatic and small intestinal fluid
secretion, and both glucagon and insulin release.

72
Q
  1. Release of which of the following peptides leads to an increase in the secretion of
    pancreatic enzymes into the small intestine?
    A. Cholecystokinin
    B. Gastrin
    C. Motilin
    D. Secretin
    E. Somatostatin
A

The correct answer is A. Release of cholecystokinin is stimulated by the presence of peptides,
amino acids, or fatty acids in the small intestine. Cholecystokinin acts on the pancreas to stimulate
secretion of pancreatic enzymes that aid in the digestion of these compounds.
Gastrin (choice B) secretion, which is stimulated by the presence of peptides or amino acids in the
lumen of the stomach, produces an increase in gastric H+ secretion.
Motilin (choice C) is a hormone that regulates the migrating myoelectric complex, a series of
contractions that occurs during fasting, clearing the stomach and small intestine of any residual food.
Secretin (choice D) secretion, which is stimulated by the presence of H+ and fatty acids in the
duodenum, causes an increase in pancreatic and biliary HCO3-release and decrease in gastric H+
release.
Somatostatin (choice E) secretion, which is stimulated by the presence of H+ in the lumen, results in
decreased release of all gastrointestinal hormones and decreased H+ secretion in the stomach.

73
Q
  1. A 60-year-old man presents with signs of hypoxemia due to hypoventilation. Which of the
    following could be the cause of hypoxemia in this patient?
    A. Anemia
    B. Fibrosis
    C. Chronic obstructive pulmonary disease (COPD)
    D. Right-to-left cardiac shunt
A

The correct answer is C.Hypoventilation is the underlying mechanism for developing hypoxemia in
the COPD patient. Decreased oxygen-carrying capacity is the underlying cause for hypoxemia in
anemia (choice A). Hypoxemia in fibrosis (choice B) is due to a V/Q mismatch. The mechanism for
hypoxemia in right-to-left cardiac shunt (choice D) is a venous admixture.

74
Q
74. The blood from an 8-year-old boy was analyzed by flow cytometry. The exact number of B
cells was counted. Which of the following cell surface markers was likely used to identify the B
cells in this blood sample?
A. CD3
B. CD4
C. CD8
D. CD19
E. CD56
A

The correct answer is D. B cells are lymphocytes capable of differentiating into the plasma cells that
produce antibodies. The best markers for identification of B cells are CD19, CD20, and CD21. The
CD21 marker is a receptor for EBV (Epstein-Barr virus).
The CD3 marker (choice A) is present on all T cells with either a CD4 or CD8 marker. This is the
marker that is used to identify total T cell count in a blood sample. The CD3 marker is used for signal
transduction in the different T cells.
The CD4 marker (choice B) is used to identify T helper cells. They are also important sites for HIV
viral infection gaining entry into these cells. These are the cells that recognize exogenous peptides
presented on MHC class II molecules by macrophages. CD4+ T helper cells are also involved in cellmediated
delayed hypersensitivity, production of cytokines for stimulation of antibody production by B
cells, and stimulation of macrophages. The CD8 marker (choice C) is used to identify cytotoxic T
cells. These are the cells that recognize viral epitopes attached to the MHC class I molecules of a
virally infected cell. The CD56 marker (choice E) is used to identify NK (natural killer) cells. These
cells are important in innate host defense, specializing in killing virally infected cells and tumor cells by
secreting granzymes and perforins.

75
Q
  1. A deficiency of the complement protein C4 would inhibit which of the following
    complement activities?
    A. Completion of the classic pathway to the splitting of C3
    B. Formation of C3b for opsonization
    C. Formation of C5 convertase by way of the alternative pathway
    D. Formation of C5a for chemotactic attractant for neutrophils
    E. Formation of the membrane attack complex
A

The correct answer is A. By definition, complement is 11 plasma proteins that interact in a chain
reaction following exposure to activated antibody surfaces at certain pathogens. This will promote cell
lysis, phagocytosis, and other defense mechanisms. The classic complement pathway involves C1,
C4, and C2 to the point where C3 splits. C3 is then split to yield C3b and C3a. A deficiency of C4
would have no effect on any of the other answer choices listed.
The complement protein C3 can be split into C3a and C3b (choice B) using the alternative pathway.
The additional proteins required in this pathway would be factors B and D and properdin. The C5
convertase enzyme can be formed in the alternative pathway (choice C) without using C4. The C5
molecule could be split into C5a (choice D) and C5b using the alternative pathway, without the
involvement of C4. The membrane attack complex (choice E; C5b, C6, C7, C8, and polymers of C9)
lyses the pathogenic cell. This process does not require the alternative pathway and would not require
C4.

76
Q
76. What substance allows platelet adhesion to exposed collagen fibers following surgery?
A. Factor VIII
B. Factor IX
C. Fibronectin
D. Tissue factor
E. Von Willebrand factor
A

The correct answer is E. Von Willebrand factor (vWF) is a self-polymerizing clotting protein present
in the serum and the subendothelial basal lamina, which has binding sites for collagen, platelets, and
fibrin. At a site of injury, vWF forms the bridge between the exposed collagen fibers and platelets in
circulation, stimulating platelet degranulation and initiating the cellular component of the clotting
cascade. An equally important role for vWF is binding platelets to the newly formed fibrin strands in a
blood clot. Factor VIII (choice A) and Factor IX (choice B) are clotting proteins of the intrinsic
pathway. Factor VIII acts in concert with activated Factor IX (IXa) to cleave Factor X to Xa. Xa is the
prothrombin activator central to both the intrinsic and extrinsic pathways. Fibronectin (choice C) is a
serum protein that acts as an opsonin for phagocytic cells in clots. Fibronectin binds non-specifically
to bacteria and other materials in the newly forming clots, and draws the cell membrane of
phagocytes into contact with these substances. Tissue factor (choice D) is a protein released from
injured tissues that works in concert with Factor VII to initiate the extrinsic pathway of coagulation.
Like Factors VIII and IX, tissue factor and Factor VII cleave Factor X to Xa.

77
Q
  1. The clearance of several substances was measured at a constant glomerular filtration rate
    and constant urine flow rate, but at increasing plasma concentrations of the substance. Under
    these conditions, clearance will increase at high plasma concentrations for which of the
    following substances?
    A. Creatinine
    B. Mannitol
    C. Penicillin
    D. Phosphate
    E. Urea
A

The correct answer is D. Clearance of a substance will change with increasing plasma concentration
if that substance is secreted or reabsorbed by a facilitated mechanism. Most phosphate is reabsorbed
by a facilitated sodium-phosphate cotransporter in the proximal tubule. As the concentration of
phosphate increases, the transporter saturates, leaving more phosphate available for excretion.

78
Q
  1. During a fast, a brief phase of intense sequential contractions begins in the stomach and
    gradually migrates to the ileum. Release of which of the following intestinal hormones is most
    likely responsible for this observed effect?
    A. Cholecystokinin
    B. Gastrin
    C. Gastrin-releasing peptide
    D. Motilin
    E. Secretin
A

The correct answer is D. Motilin is a hormone released by the small intestine during the fasting
state. Its waxing and waning blood levels correlate with the initiation and ending of migrating motor
complexes (MMC). Furthermore, injection of motilin has been shown to evoke MMC activity. The
MMC typically begins in the stomach, and over a 90-120 minute period, migrates to the ileum, where it
dies out. As one complex dies out in the ileum, another complex begins in the stomach provided the
fasting state continues. Eating a meal interrupts the MMC activity.
Cholecystokinin (choice A) is released during the intestinal phase of the digestive period (not during a
fast). Its secretion is evoked by the presence of fat and protein digestion products in the duodenum. It
induces contraction of the gall bladder and relaxation of the sphincter of Oddi.
Gastrin (choice B) is released from G cells in the antrum, mostly during the gastric phase of the
digestive period (not during a fast). It tends to increase stomach motility, although the rate of emptying
is decreased because gastrin also causes the pyloric sphincter to contract. It also may contribute to
the increase in ileal and colonic motility as part of the gastroileal and gastrocolic reflexes, respectively.
Gastrin-releasing peptide (choice C) mediates the neural release of gastrin. Antral enteric neurons
that are activated by vagal efferents or by local reflexes release gastrin-releasing peptide, which
stimulates the G cells to secrete gastrin. Secretin (choice E) is a duodenal hormone that is released
during the intestinal phase of the digestive period (not during a fast). Its secretion is evoked by a
duodenal pH less than 4.5. Secretin tends to decrease the rate of stomach emptying.

79
Q
  1. Which of the following conditions is associated with a lifelong requirement for
    administration of parenteral vitamin B12?
    I. Pernicious anemia II. Removal of colon III. Removal of gallbladder IV. Removal of ileum V.
    Removal of jejunum VI. Removal of stomach
    A. I only
    B. I and V
    C. I, IV, and V
    D. I, IV, and VI
    E. III only
A

The correct answer is D. This question tests your knowledge of how vitamin B12 is normally
absorbed. In summary, parietal cells in the gastric lining secrete a glycoprotein called intrinsic factor
into the gastric lumen. This protein binds to vitamin B12, protecting it from degradation and allowing
for its eventual absorption. At the level of the ileum, B12 bound to intrinsic factor is actively
reabsorbed. Therefore, a loss of intrinsic factor or of its reabsorption site, the ileum, would lead to the
need for lifelong injection of vitamin B12. Removal of the stomach would obviously lead to loss of
intrinsic factor. Pernicious anemia is actually an autoimmune disease that targets the gastric
epithelium. In many cases, patients have autoantibodies against intrinsic factor, preventing the
absorption of ingested vitamin B12.
Removal of the colon, jejunum, or gallbladder would not affect reabsorption of intrinsic factor or B12.
A total absence of bile (which occurs with bile duct blockage, but not with cholecystectomy) may lead
to malabsorption of fat-soluble vitamins such as A, D, E, and K, but not of B12.

80
Q
  1. Which of the following procoagulant proteins binds to tissue factor exposed beneath the
    endothelium of traumatized blood vessels and initiates the clotting cascade?
    A. Factor V
    B. Factor VII
    C. Factor X
    D. Fibrinogen
    E. Prothrombin
A

The correct answer is B. The extrinsic pathway of clotting begins with tissue factor binding to Factor
VII or Factor VIIa. All other clotting proteins require proteolytic cleavage to become active; however,
Factor VII has a low level of activity in its inactive form and can act with tissue factor and
phospholipids to initiate the clotting cascade. In the extrinsic pathway, Factor VII cleaves Factor X to
Xa (choice C), which acts in concert with Factor V (choice A) to cleave prothrombin to thrombin
(choice E). The final step in the coagulation pathway is the cleavage of fibrinogen to fibrin by
thrombin (choice D). Fibrin polymerizes and cross-links, thereby forming a hemostatic net of insoluble
protein.

81
Q
  1. Which sequence of the numbered statements below correctly describes the pathway of
    lymphatic flow on the left side of the body?
    1) Junction of the left internal jugular and left subclavian 2) Lymph capillaries 3) Thin lymph
    vessels 4) Thoracic duct
    A. 2-1-3-4
    B. 2-3-1-4
    C. 2-3-4-1
    D. 2-4-1-3
    E. 2-4-3-1
A

The correct answer is C. On the left side of the body, the lymphatic fluid flows from the lymphatic
capillaries to the thin lymphatic vessels and then to the thoracic duct, which empties into the junction
of the left internal jugular and left subclavian veins. On the right side, the lymphatic fluid flows from the
lymphatic capillaries to the thin lymphatic vessels to the right thoracic duct, which empties into the
junction of the right internal jugular and the right subclavian veins.

82
Q
  1. The oxygen-hemoglobin dissociation curve shifts to the left under which of the following
    circumstances?
    A. Carbon monoxide poisoning
    B. Decreased pH
    C. Increased 2,3-diphosphoglycerate (2,3-DPG)
    D. Increased PCO2
    E. Increased temperature
A

The correct answer is A. The loading of O2 is facilitated when the oxygen dissociation curve shifts
to the left, and the unloading of O2 is facilitated when the oxygen dissociation curve shifts to the right.
Carbon monoxide (CO) poisoning is extremely dangerous for several reasons. CO left-shifts the
oxygen dissociation curve, which interferes with the unloading of O2. Also, CO, which has
approximately 240 times the affinity for hemoglobin than O2 does, preferentially binds to available
sites on hemoglobin.
The remaining answer choices all shift the oxygen-hemoglobin dissociation curve to the right. A good
way to remember the conditions that promote dissociation of O2 is to think of exercising muscle,
which has decreased pH (choice B) from lactic acid buildup and increased PCO2 (choice D),
increased 2,3-DPG (2,3-diphosphoglycerate; choice C) because of increased glycolysis, and
increased temperature (choice E).

83
Q
  1. A patient has recurrent attacks of asthma that occur most frequently after aspirin
    administration. Which of the following is the pathogenetic mechanism in this form of asthma?
    A. Direct release of bronchoconstrictor mediators
    B. Enhanced sensitivity to vagal stimulation
    C. Inhibition of cyclooxygenase pathway
    D. Type I hypersensitivity reaction
    E. Type IV hypersensitivity reaction
A

The correct answer is C. Aspirin-induced asthma is an infrequent form of asthma. It is related to the
direct pharmacologic action of aspirin on the metabolism of arachidonic acid. Aspirin inhibits the
cyclooxygenase pathway without affecting the lipoxygenase pathway, leading to a decreased ratio of
prostaglandins (bronchodilators) to leukotrienes (bronchoconstrictors). The disrupted balance
between these two groups of arachidonic acid metabolites leads to bronchoconstriction in
predisposed patients.
Direct release of bronchoconstrictor substances (choice A) is one of several pathogenetic
mechanisms that may mediate occupational asthma, triggered by inhalation of several chemicals,
including epoxy resins, plastics, cotton fibers, toluene, formaldehyde, and penicillin products.
Enhanced vagal stimulation (choice B) plays an essential role in nonatopic asthma. This variety of
asthma is initiated by viral infections of the upper respiratory tract (e.g., common cold and flu), which
appear to lower the threshold of respiratory mucosa to parasympathetic (vagal) stimulation and its
bronchoconstrictor influence on the lungs.
Type I hypersensitivity reactions (choice D) are crucial in the pathogenesis of atopic asthma following
prior exposure to several allergens. T-cell activation instructs B cells to produce IgE directed against a
given allergen. On reexposure, IgE on the surface of mast cells binds the allergen and induces mast
cell degranulation. The mediators released from mast cells cause bronchospasm and recruit more
inflammatory cells, including eosinophils, lymphocytes, and basophils. Type IV hypersensitivity
reactions (choice E) do not occur in asthma.

84
Q

84.The hormone that is essential for nervous tissue maturation, ossification of bone,
conversion of carotene to vitamin A, and increasing absorption of glucose from the small
intestine is:
A. a peptide hormone released from the parathyroid
B. formed from iodine, peroxide, and thyroglobin
C. secreted by alpha cells of pancreatic islets
D. secreted from the adrenal medulla

A

The correct answer is choice B. Thyroid hormone is a major anabolic hormone essential for normal
growth and maturation. It is synthesized from iodine, peroxidase, and thyroglobulin through oxidation,
iodination, and coupling. Choice A Parathyroid hormone increases calcium reabsorption and
decreases phosphate reabsorption, thus increasing free calcium level. Choice C Glucagon is
secreted by alpha cells of pancreatic islets. It increases serum glucose through liver glycogenolysis
and gluconeogenesis. Choice D Most of the adrenal medulla secretion is epinephrine and some
norepinephrine. It increases glucose output by liver and muscle glycogenolysis and increased lipolysis
in adipose tissue.

85
Q
85. In which portion of the kidney tubule is most water normally reabsorbed?
A. Cortical collecting duct
B. Distal tubules
C. Loops of Henle
D. Medullary collecting duct
E. Proximal tubule
A

The correct answer is E.choice C), 10% in the distal tubules (choice B), and less than 10% in the
collecting ducts (choices A and D); approximately 1 liter of water is normally excreted as urine each
day. The amount of water reabsorbed in the proximal tubule and loop of Henle is not affected by ADH,
because ADH does not affect tubular permeability in these segments of the nephron. ADH increases
the permeability of the distal tubules and collecting duct, however, which increases reabsorption of
water. When ADH levels are high, the urine output can decrease to less than 0.5 L/day; when ADH
levels are low, the output of urine can increase to more than 30 L/day. Even at these extremes,
however, most of the water in the glomerular filtrate is still reabsorbed in the proximal tubule.

86
Q
  1. Which of the following series of test results will confirm that a woman is postmenopausal?
    A. Decreased LH, decreased FSH, increased estrogen
    B. Decreased LH, increased FSH, decreased estrogen
    C. Increased LH, decreased FSH, decreased estrogen
    D. Increased LH, increased FSH, decreased estrogen
    E. Increased LH, increased FSH, increased estrogen
A

The correct answer is D. During menopause there is a loss of functioning follicles in the ovaries such
that GnRH-stimulated LH and FSH secretion do not result in normal estrogen secretion. The low
estrogen levels cannot inhibit gonadotropin secretion in a negative-feedback fashion, resulting in very
high levels of LH and FSH. Choices A, B, C, and E do not accurately describe normal levels in
menopause.

87
Q
  1. Cancer of which of the following organs is the most likely cause of an elevation of serum
    erythropoietin levels in a patient with a hematocrit of 52%?
    A. Breast
    B. Colon
    C. Kidney
    D. Stomach
    E. Thyroid
A

The correct answer is C. This pathophysiology question is essentially asking, “Which of the following
organs produces erythropoietin?” The kidney normally produces erythropoietin, which stimulates the
erythroid cell line in the bone marrow. Renal cell carcinoma can lead to overproduction of
erythropoietin and thereby cause secondary polycythemia. Other causes of secondary polycythemia
are diseases that impair oxygenation, including pulmonary diseases (including smoking) and
congestive heart failure. Breast cancer (choice A) can present as an axillary mass or as Paget
disease of the nipple. Colon cancer (choice B) and stomach cancer (choice D) can present with
anemia secondary to blood loss. Thyroid cancer (choice E) can present with hypothyroidism.

88
Q
  1. In the clotting process, as the hemostatic plug develops, fibrin polymerizes into
    monomeric threads that are held together by noncovalent bonds. Which clotting protein
    increases the strength of the clot by cross-linking the newly formed fibrin threads?
    A. Factor XIII
    B. High molecular weight kininogen (HMWK)
    C. Plasminogen
    D. Thrombin
    E. von Willebrand factor (vWF)
A

The correct answer is A. Fibrinogen is cleaved by thrombin twice as it is activated to form fibrin. The
initial cleavage causes it to polymerize, and the second causes it to branch. Thrombin also activates
Factor XIII and XIIIa, which cross-link the fibrin strands and strengthen the clot. High molecular weight
kininogen (HMWK; choice B) is a cofactor in the intrinsic pathway that converts Factor XI to XIa.
Plasminogen (choice C) is a central proenzyme in clot lysis. When plasminogen is converted to
plasmin, it digests fibrin threads and several protein factors, including Factors V, VIII, XII, and
prothrombin. Thrombin (choice D) is an enzyme derived from prothrombin. It converts fibrinogen to
fibrin and activates Factor XIII. von Willebrand Factor (vWF; choice E) is a tissue-bound protein that
is exposed with vascular trauma and helps in the process of platelet adhesion.

89
Q
  1. Which of the following agents affecting neuromuscular transmission acts by blocking
    release of acetylcholine (ACH) from presynaptic terminals?
    A. Curare
    B. Neostigmine
    C. Hemicholinium
    D. Botulinus toxin
A

The correct answer is D. Botulinus toxin causes total blockade by blocking the release of ACH from
the presynaptic terminal. Curare (choice A) decreases the size of EPP by competing with ACH for
receptors on the motor endplate. Neostigmine (choice B) prolongs action of ACh, acting as an
anticholinesterase inhibitor. Hemicholinium (choice C) depletes ACH stores from the presynaptic
terminal.

90
Q
90. During an exercise stress test, which of the following is most likely to occur in the skeletal
muscles?
A. Decreased blood flow
B. Decreased metabolite concentrations
C. Increased arteriolar diameter
D. Increased oxygen concentration
E. Increased vascular resistance
A

The correct answer is C. Blood flow can increase as much as 20-fold in exercising skeletal muscle,
which is a greater increase than in any other tissue in the body. This tremendous increase in blood
flow results almost entirely from the actions of local vasodilator substances on the muscle arterioles.
During exercise, the muscles use oxygen more rapidly than it can be delivered by the blood, which
decreases the oxygen concentration (choice D) in the tissues. The oxygen deficiency causes
vasodilator metabolites (choice B) such as adenosine, carbon dioxide, lactic acid, and others to
accumulate in the tissues. The vasodilator metabolites acting on the arterioles lead to a reduction in
vascular resistance (choice E) and an increase in blood flow (choice A).

91
Q
  1. In the transition from a Graafian follicle to a functional corpus luteum, which of the
    following cellular events occurs?
    A. Granulosa cells begin to express aromatase
    B. Granulosa cells begin to express FSH receptors
    C. Granulosa cells begin to express LH receptors
    D. Theca cells begin to express LH receptors
    E. Theca cells begin to express side-chain cleavage enzyme
A

The correct answer is C. The secretion of estrogen by the developing follicle can best be explained
using the “two cell” hypothesis. Theca cells are stimulated by LH (theca cells express LH receptors
before formation of the corpus luteum, choice D) to secrete the androgens androstenedione and
testosterone. The androgens then diffuse into the granulosa cells, where they are aromatized to
estrogens. Hence, theca cells express side-chain cleavage enzyme (first step in steroidogenesis)
before the formation of the corpus luteum (choice E). FSH stimulates aromatase activity in the
granulosa cells (receptors for FSH and aromatase enzyme are present before the formation of the
36
corpus luteum, choices A and B). The granulosa cells apparently have the ability to produce steroids
(progesterone), but lack 17-hydroxylase activity and cannot synthesize estrogen themselves. Only as
the follicle approaches ovulation do LH receptors begin to be expressed by the granulosa cells.
Estrogen and FSH probably are responsible for the change. After ovulation, the scar of the follicle
undergoes luteinization. The theca cells decrease 17-hydroxylase activity and secrete more
progesterone. The granulosa cells decrease aromatase activity and also secrete more progesterone.

92
Q
  1. What role do the lungs play in the pathogenesis of systemic hypertension?
    A. The alveolar capillaries contain aldosterone
    B. The alveolar capillaries contain angiotensin-converting enzyme (ACE)
    C. The alveolar capillaries contain antidiuretic hormone (ADH)
    D. The type II pneumocytes contain aldosterone
    E. The type II pneumocytes contain angiotensin-converting enzyme (ACE)
A

The correct answer is B. Renin is a renal protease that is produced when there is decreased renal
perfusion. Renin then cleaves angiotensinogen (secreted into the plasma by the liver) to form
angiotensin I. Angiotensin I is cleaved by angiotensin-converting enzyme (found in very high
concentrations in the endothelial cells of the pulmonary capillary bed) to form angiotensin II.
Angiotensin II triggers release of aldosterone from the adrenal cortex. Aldosterone increases Na+
resorption from the cortical collecting duct of the kidney, thereby tending to increase blood volume
that in turn increases blood pressure. Under normal circumstances, angiotensin-converting enzyme is
present in large amounts and is not rate-limiting in this process; however, it does provide an effective
point at which the pathway can be interrupted by ACE inhibitors. Physiologically, the ACE inhibitors
primarily inhibit the renin-angiotensin-aldosterone system. These agents also inhibit bradykinin
degradation and stimulate vasodilating prostaglandin synthesis.
Aldosterone (choices A and D) is produced in the adrenal cortex. This hormone stimulates sodium
and water conservation in the kidneys.
ADH (choices C and E) is secreted by the posterior pituitary. This hormone decreases water
secretion in the kidneys.
Angiotensin-converting enzyme is present in the endothelial cells of the alveolar capillary bed, not in
the type II pneumocytes (choice E).

93
Q
93. Which of the following would be expected in a woman with isolated ACTH-deficiency?
A. Decreased pubic and axillary hair
B. Decreased serum sodium concentration
C. Hyperpigmentation
D. Increased serum cortisol
E. Increased serum potassium
A

The correct answer is A. Isolated ACTH-deficiency is a pituitary disorder characterized by
decreased secretion of ACTH, but not of the other hormones of the anterior pituitary. Because of the
decreased effect on the inner two zones of the adrenal cortex, the secretion of adrenal androgens is
decreased. Because pubic and axillary hair in females depends on adrenal androgens, ACTH
deficiency would lead to a decrease in both. Like primary adrenal insufficiency, isolated ACTHdeficiency
would lead to low serum levels of cortisol (not increased, choice D). With lower levels of
ACTH and cortisol, patients may also present with signs and symtoms of Addison disease, such as
low blood pressure, dehydration, and nausea. Unlike primary adrenal insufficiency, however, serum
levels of aldosterone are usually unchanged with isolated ACTH deficiency. This is because ACTH is
not a long-term regulator of aldosterone secretion (angiotensin II and potassium are the main longterm
regulators). Because aldosterone is unchanged, serum sodium (choice B) and serum potassium
(choice E) are normal. Hyperpigmentation (choice C) is associated with increased, not decreased,
serum levels of ACTH.

94
Q
  1. Which of the following is most likely to be decreased during resting conditions for a
    patient that has been severely anemic (Hb = 6.0 g/dL) for several months?
    A. 2,3-diphosphoglycerate
    B. Arterial PO2
    C. Cardiac output
    D. Mixed venous PO2
    E. Red blood cell H+ concentration
A

The correct answer is D. Oxygen delivery to the tissues is approximately equal to cardiac output x
hemoglobin concentration x the amount of oxygen extracted from the blood (oxygen extraction =
arterial oxygen content - venous oxygen content). Some of the compensatory mechanisms that come
into play in an anemic person with a low hemoglobin content include the following: increased levels of
2,3 diphosphoglycerate (2,3-DPG; choice A), increased cardiac output (choice C), and increased red
blood cell H+ concentration (choice E). The increase in 2,3-DPG and red blood cell H+ concentration
cause the oxygen-hemoglobin dissociation curve to shift to the right, which facilitates unloading of
oxygen in the tissues.
The arterial PO2(choice B) and arterial oxygen saturation of hemoglobin are independent of
hemoglobin concentration. A decrease in the hemoglobin concentration of the blood, however, causes
a proportionate decrease in the oxygen-carrying capacity of the blood. (Think of a glass beaker
containing a solution of hemoglobin. If a gas having a PO2 of 100 mm Hg is bubbled through the
solution, the PO2 of the solution will be 100 mm Hg, regardless of the hemoglobin concentration. If
the concentration of hemoglobin in the beaker is doubled, however, the beaker will now contain twice
as much oxygen still at 100 mmHg.) Because each gram of hemoglobin can normally carry a total of
1.34 grams of oxygen, the arterial oxygen content of this patient can be calculated as follows: 1.34 mL
O2/g Hb x 6.0 g Hb/dL blood = 8.04 mL O2/dL blood (normal = 20 mL O2/dL blood). When the oxygen
content of the arterial blood is decreased to only 8 mL O2 /dL, the loss of oxygen to the tissues will
cause the venous oxygen tension to decrease to lower than normal levels during rest and to very low
levels whenever exercise is attempted.

95
Q
95. Which of the following regions of the nephron is capable of the greatest level of glucose
reabsorption?
A. Proximal convoluted tubule
B. Thick ascending limb of loop of henle
C. Distal convoluted tubule
D. Collecting duct
A

The correct answer is A.At plasma concentrations less than 250 mg/dL, all of the filtered glucose is
reabsorbed in the proximal convoluted tubule and excretion is zero. The point at which the carriers are
saturated is Tm, that is, plasma concentration greater than 350 mg/dL. As the plasma concentration
increases, the additional filtered glucose cannot be reabsorbed and is excreted in the urine.
Glucosuria is an important clinical clue to diabetes mellitus.

96
Q
96. Which of the following cytokines is associated with the development of cell-mediated
immunity?
A. IL-3
B. IL-4
C. IL-5
D. IL-10
E. IL-I2
A

The correct answer is E. Interleukins are peptides released by activated monocytes and
lymphocytes that assist in the coordination of cellular and humoral responses. Gamma interferon from
T helper 1 CD4+ lymphocytes and IL-12 from macrophages are major stimulators of the development
of a cell-mediated immune response to intracellular organisms such as tuberculosis, leishmaniasis,
histoplasmosis, and many other organisms.
38
IL-3 (choice A) is a cell growth factor produced by thymic epithelial cells and T cells to regulate
different T cell differentiation processes. IL-4 (choice B) is produced by T helper 2 CD4+ lymphocytes
and is involved in class switching to IgE. It also downregulates T helper 1 CD4+ lymphocytes. IL-5
(choice C) is produced by T helper 2 CD4+ lymphocytes and is involved in IgA production, eosinophil
differentiation, and B cell differentiation. IL-10 (choice D) is produced by T helper 2 CD4+
lymphocytes. It downregulates the cell-mediated immune system.

97
Q
  1. Which of the following pairs of hormones are most important to replace immediately after
    removal of a tumor near the sella turcica, before life-threatening symptoms develop due to
    anterior pituitary insufficiency?
    A. Estrogens and mineralocorticoids
    B. Glucocorticoids and thyroid hormones
    C. Growth hormone and luteinizing hormone
    D. Prolactin and glucocorticoids
    E. Testosterone and follicle stimulating hormone
A

The correct answer is B. The anterior pituitary produces the following hormones: thyroid stimulating
hormone (TSH), adrenocorticotropic hormone (ACTH), follicle stimulating hormone (FSH), leutinizing
hormone (LH), prolactin, and growth hormone. Although deficiencies of any of these hormones are
serious, the most dangerous hormonal deficiencies involve glucocorticoids and thyroid hormone. It
has consequently become customary to “cover” patients for these potential deficiencies immediately
after any event (including surgery and possible pituitary apoplexy) in which function of the anterior
pituitary may be lost. If it turns out that the function was not lost, these hormones can be discontinued
later.
Deficiency of vasopressin produces decreased mineralocorticoid release (choice A), and clinically
recognizable diabetes insipidus (high urinary output with inability to concentrate urine), but
replacement is usually not begun unless symptoms become apparent.
Gonadal steroids (not LH and FSH) are given eventually, but they are not urgently required (choices
A, C, and E).

98
Q
  1. Which of the following is most likely to be decreased in the skeletal muscles of the legs of
    a patient with essential hypertension following a severe laceration on the right hand?
    A. Adenosine levels
    B. Arterial blood pressure
    C. Arteriolar resistance
    D. Blood flow
    E. Venous oxygen concentration
A

The correct answer is A. The skeletal muscles of the body have a normal blood flow even when the
blood pressure is chronically elevated. Organs and tissues in which the vasculature has primarily a
nutritive function (e.g., brain, heart, and skeletal muscle) regulate their blood flow in accordance with
the metabolic needs of the tissues. These tissues exhibit short-term autoregulation of blood flow such
that the increase in flow caused by an elevated arterial pressure is minimized by constriction of the
arterioles. The constriction is caused in part by decreased levels of adenosine (an endogenous
vasodilator) in the tissues. The rate of adenosine production in a tissue is a function of its metabolic
rate, which is not affected significantly by an increase in systemic pressure. When blood flow to the
muscle increases, the adenosine is literally washed from the muscle, lowering the tissue levels of
adenosine. The decrease in adenosine concentration causes small arteries and arterioles in the
muscle to constrict, and this increase in resistance (choice C) maintains blood flow (choice D) at a
normal rate in the face of increased arterial pressure (choice B). The overall process is called
autoregulation of blood flow.
Venous oxygen concentration (choice E) does not decrease in the skeletal muscles of patients with
hypertension because blood flow is maintained at an adequate level to meet the nutritional demands
of the muscles.

99
Q
  1. Which of the following would be present in a patient with a small-cell carcinoma of the
    lung that autonomously secretes vasopressin?
    A. Blood volume contraction
    B. Decreased plasma atrial natriuretic peptide
    C. Hypernatremia
    D. Inappropriately concentrated urine
    E. Increased thirst
A

The correct answer is D. A tumor that ectopically secretes vasopressin can produce a disorder
called the syndrome of inappropriate antidiuretic hormone (SIADH). Hypovolemia physiologically
stimulates ADH (vasopressin) secretion, so the diagnosis of SIADH is made only if the patient is
euvolemic. A hallmark of this disorder is excessive renal retention of free water with resultant
hypervolemia, not blood volume contraction (choice A), and dilutional hyponatremia, not
hypernatremia (choice C). The volume expansion leads to increased, not decreased (choice B),
secretion of atrial natriuretic peptide (ANP) by atrial myocytes. The increased ANP is one of the
factors that causes the kidney to increase sodium excretion and produce an inappropriately
concentrated urine. Urine is typically hypertonic to plasma in this disorder. The volume expansion also
tends to suppress renin secretion, and the resultant decrease in serum aldosterone may also
contribute to the increased renal sodium excretion. The dilutional hypotonicity would inhibit, not
stimulate (choice E), hypothalamic thirst mechanisms.

100
Q
  1. On stimulating the afferent portions of the glossopharyngeal and vagus nerves, which of
    the following outcomes would most likely occur?
    A. Bradycardia with hypertension
    B. Bradycardia with hypotension
    C. Sinus arrhythmia with hypotension
    D. Tachycardia with hypertension
    E. Tachycardia with hypotension
A

The correct answer is B. The glossopharyngeal nerve (CN IX) and the vagus nerve (CN X) carry
afferent information to the medulla from the carotid sinus and aortic arch baroreceptors, respectively.
The firing rate of these neurons increases with increasing blood pressure. By artificially increasing the
firing rate of these nerves, the medulla therefore receives a false signal that indicates that the blood
pressure is too high. This elicits a baroreceptor reflex, resulting in a decrease in sympathetic outflow
and an increase in parasympathetic outflow, which leads to bradycardia and hypotension.
Physiologically the glossopharyngeal nerve innervates the sensory nuclei of the medulla oblongata.
The somatic motor portion innervates the muscles involved with swallowing. The visceral motor
portion affects the parotid salivary gland by way of the otic ganglion. The vagus nerve innervates the
sensory fibers to sensory nuclei and autonomic centers of the medulla oblongata. The visceral motor
fibers go to muscles of the palate, pharynx, digestive, respiratory, and cardiovascular systems in the
thoracic and abdominal cavities.

101
Q
101. Which of the following controls the salivary secretion?
A. Parasympathetic nervous system
B. Gastrin
C. Secretin
D. CCK
A

The correct answer is A. Salivary secretion is almost entirely under control of the parasympathetic
system. Sympathetic stimulation causes transient minor stimulation. Parotid secretion is stimulated by
parasympathetic stimulation from CN IX, whereas submandibular and sublingual secretion is
stimulated by parasympathetic stimulation from CN VII. Choice B - Gastric secretion is stimulated by
gastrin. Choice C - Pancreatic secretion is stimulated by secretin. Choice D - CCK stimulates bile
secretion.

102
Q
  1. Erythropoietin stimulates which of the following intermediates in hematopoiesis?
    A. Basophilic erythroblasts
    B. Colony forming units-erythroid (CFU-E)
    C. Multipotential stem cells
    D. Proerythroblasts
    E. Reticulocytes
A

The correct answer is B. The colony forming unit-erythroid (CFU-E) is a unipotential stem cell that
develops from a burst forming unit-erythroid (BFU-E), which develops eventually from the
multipotential stem cell. The BFU-E is somewhat responsive to erythropoietin, but the CFU-E is
completely dependent on erythropoietin. Erythropoietin is normally released from the kidney in
response to hypoxic or anemic conditions.
The basophilic erythroblast (choice A) differentiates from the proerythroblast. It is recongnizable by
light microscopy and has a dark basophilic staining due to hemoglobin synthesis. It is not directly
affected by erythropoietin, but is instead indirectly increased by the increase in precursor cells from
the increase in CFU-E earlier in development.
The multipotential stem cell (choice C) appears earlier in development than CFU-E and does not
increase with erythropoietin. The development of all major components of blood (RBC, WBC, and
platelets) begins with the multipotential stem cell (CFU-S). This cell is noncommitted and can selfrenew.
It is located in the bone marrow and is not recognizable by light microscopy.
The proerythroblast (choice D), which is the first recognizable cell in the red cell lineage, develops
from the CFU-E cell. It is not affected directly by erythropoietin, but instead increases in number from
the increased CFU-E cells.
The reticulocyte (choice E) is the enucleated cell just before the mature red blood cell. Reticulocytes
enter the peripheral circulation but continue to synthesize hemoglobin. This cell is not directly
stimulated by erythropoietin, but increases in number as a result of the increase in precursors.

103
Q
  1. In a controlled experiment, radiolabeled ATP is injected into an isolated muscle. The
    muscle is stimulated and allowed to contract for 10 seconds. An autoradiogram from a biopsy
    of the muscle will show radiolabeled ATP bound to
    A. actin
    B. myosin
    C. sarcoplasmic reticulum
    D. tropomyosin
    E. troponin C
A

The correct answer is B. During the contraction cycle, ATP binds to myosin, causing the dissociation
of myosin from actin. Actin (choice A) forms cross-bridges with myosin but does not bind to ATP. The
sarcoplasmic reticulum (choice C) is involved in storing and releasing Ca2+ for muscle contraction.
Tropomyosin (choice D) is a thin filament that runs alongside actin. In the absence of calcium,
tropomyosin lies in the groove of the actin filament and blocks actin’s myosin-binding sites. Troponin
C (choice E) is the calcium-binding subunit of the troponin complex. When troponin C binds calcium,
a conformational change causes tropomyosin to shift, thereby exposing the myosin-binding sites on
actin.

104
Q
104. Which of the following would shift the oxygen-hemoglobin dissociation curve to the
right?
A. Carbon monoxide poisoning
B. Decreased PCO2
C. Decreased pH
D. Decreased temperature
E. Decreased 2,3-DPG
A

The correct answer is C. The loading of O2 is facilitated when the oxygen dissociation curve shifts to
the left, and the unloading of O2 is facilitated when the oxygen dissociation curve shifts to the right. A
good way to remember the conditions that promote dissociation of O2 is to think of exercising muscle,
which has decreased pH (choice C) because of the accumulation of lactic acid, increased PCO2
(compare with choice B) because of the increased rate of aerobic metabolism, increased temperature
(compare with choice D), and increased 2,3-DPG (2,3-diphosphoglycerate; compare with choice E)
because of increased glycolysis.
Carbon monoxide poisoning (choice A) left-shifts the oxygen dissociation curve, which interferes with
the unloading of O2. Carbon monoxide also strongly binds to available sites on hemoglobin.

105
Q
105. After a total gastrectomy, which of the following digestive enzymes will be produced in
inadequate amounts?
A. Amylase
B. Chymotrypsin
C. Lipase
D. Pepsin
E. Trypsin
A

The correct answer is D. Gastrectomy is removal of the entire stomach. Pepsin is secreted (in an
inactive, or zymogen, form, as pepsinogen) by the chief cells of the stomach. Pepsinogen is activated
by contact with stomach acid. Although protein digestion usually begins with the actions of
hydrochloric acid and pepsin, pancreatic enzymes complete the job as the food passes into the small
intestine. Amylases (choice A) hydrolyze 1, 4 glycosidic linkages of starches to produce
oligosaccharides, maltose, maltotriose, and dextrins. Amylases are produced by the pancreas and
salivary glands. Chymotrypsin (choice B) is a proteolytic enzyme released by the pancreas as the
inactive proenzyme, chymotrypsinogen. Lipases (choice C) are mostly released by the pancreas and
serve to digest various lipids, including triacylglycerols. Trypsin (choice E) is a proteolytic enzyme
released by the pancreas as the inactive proenzyme trypsinogen.

106
Q
106. Which of the following cardiovascular changes is most likely to occur when changing
from a standing to a supine position?
A. Decreased myocardial contractility
B. Decreased total peripheral resistance
C. Dilation of large veins
D. Increased pulse
E. Increased renal blood flow
A

The correct answer is D. Baroreceptors detect pressure changes in the walls of blood vessels and in
portions of the digestive, reproductive, and urinary tracts. The baroreceptor mechanism is important
for maintaining arterial pressure when a person sits or stands from a lying position. When a person
suddenly stands, the blood pressure in the brain and upper body tends to fall, which initiates a strong
sympathetic discharge throughout the body aimed at returning blood pressure to normal. Increasing
sympathetic stimulation to the heart causes an increase in heart rate, conduction velocity, and
myocardial contractility (compare with choice A). The sympathetic stimulation also causes
constriction of nearly all the arterioles in the body, which greatly increases the total peripheral
resistance (compare with choice B). Sympathetic stimulation of the renal vasculature leads to a
decrease in renal blood flow (compare with choice E). Constriction of large veins (compare with
choice C) increases venous return to the heart, causing the heart to pump increased amounts of
blood.

107
Q
  1. A man competing in a weightlifting competition lifts 325 lb over his head and holds it
    there for 5 seconds. Suddenly, his arms give way and he drops the weight to the floor. Which
    of the following receptors is responsible for this sudden muscle relaxation?
    A. Free nerve endings
    B. Golgi tendon organ
    C. Merkel disk
    D. Muscle spindle
    E. Pacinian corpuscle
A

The correct answer is B. Normally, stretching of muscle results in a reflex contraction: the harder the
stretch, the stronger the contraction. At a certain point, when the tension becomes too great, the
contracting muscle suddenly relaxes. The reflex that underlies this sudden muscle relaxation is called
the Golgi tendon organ (GTO) reflex, also known as the inverse stretch reflex or autogenic inhibition.
The GTO is an extensive arborization of nerve endings that is connected in series with the extrafusal
skeletal muscle fibers. As a result, GTOs respond to muscle tension rather than muscle length.
Increased tension leads to stimulation of Ib afferents, which inhibit the homonymous muscle by way of
spinal interneurons. Free nerve endings (choice A) are unmyelinated, unencapsulated nerve endings
that penetrate the epidermis. These types of receptors respond to pain and temperature. Merkel disks
(choice C) are composed of specialized tactile epidermal cells and their associated nerve endings.
42
They are located in the basal layer of the epithelium and are slowly adapting receptors that respond to
touch and pressure.
Muscle spindles (choice D) are spindle-shaped bundles of muscle fibers (intrafusal fibers) that are
arranged in parallel with extrafusal skeletal muscle fibers, so they sense the length of the muscle.
They are innervated by Group Ia and II sensory afferent neurons.
Pacinian corpuscles (choice E) are unmyelinated nerve endings surrounded by thin, concentric layers
of epithelioid fibroblasts. In transverse section, this receptor resembles a sliced onion. They are found
primarily in the deep layer of the dermis, loose connective tissue, male and female genitalia,
mesentery, and visceral ligaments. They are rapidly adapting receptors that respond to touch and
pressure.

108
Q
  1. During the early stages of infection, which of the following compounds exert the most
    powerful chemotactic effect on neutrophils, causing them to migrate into an inflamed area?
    A. C5a and IL-8
    B. IL-1 and tumor necrosis factor
    C. LTC4 and LTD4
    D. PGI2 and PGD2
    E. Thromboxane and platelet activating factor
A

The correct answer is A. The most important chemotactic factors for neutrophils are the complement
factor C5a and the interleukin IL-8. The cytokines IL-1 and tumor necrosis factor (choice B) have
complex, similar actions, including stimulation of production of many acute-phase reactions,
stimulation of fibroblasts, and stimulation of endothelium. Leukotrienes LTC4 and LTD4 (choice C)
cause increased vascular permeability. Prostaglandins PGI2 and PGD2 (choice D) mediate
vasodilation and pain. Thromboxane and platelet activating factor (choice E) induce platelet changes.

109
Q
109. Which of the following is decreased as a result of hyperventilation?
A. Arterial oxygen content
B. Arterial oxygen tension (PO2)
C. Arterial pH
D. Cerebral blood flow
E. Cerebrovascular resistance
A

The correct answer is D. The key symptom is hyperventilation, which occurs if the rate and depth of
respiration exceeds the demands for oxygen delivery and carbon dioxide removal. Hyperventilation
results in hypocapnia, alkalosis, increased cerebrovascular resistance, and decreased cerebral blood
flow. Carbon dioxide plays an important role in the control of cerebral blood flow. An increase in
arterial PCO2 dilates blood vessels in the brain and a decrease in PCO2 causes vasoconstriction.
The anxious, hyperventilating woman is “blowing off” carbon dioxide, which lowers her arterial PCO2,
causing the cerebrovascular resistance (choice E) to increase, thereby decreasing cerebral blood
flow. The decrease in cerebral blood flow has caused the woman to feel faint and to have blurred
vision. Other symptoms commonly associated with the hyperventilation of anxiety states are feelings
of tightness in the chest and a sense of suffocation.
Hyperventilation increases the arterial oxygen content (choice A) and PO2(choice B) in a normal
person.
A decrease in arterial PCO2 causes the arterial pH (choice C) to increase (i.e., the patient becomes
alkalotic).

110
Q
  1. Following electrical stimulation of the raphe nuclei, which of the following
    neurotransmitter levels would be expected to increase?
    A. Acetylcholine
    B. Dopamine
    C. Gamma-amino-butyric acid (GABA)
    D. Norepinephrine
    E. Serotonin
A

The correct answer is E. Serotonin is the primary neurotransmitter of the raphe nuclei. Ascending
serotonin projections from the dorsal and median raphe nuclei distribute diffusely throughout the
brain. Other raphe nuclei provide descending serotonin projections to the spinal cord and brainstem.
Cell bodies that contain acetylcholine (choice A) are found in the basal nucleus of Meynert (which
degenerates in Alzheimer disease), in the medial septum and diagonal band complex, and in the
striatum.
Dopamine- (choice B) containing cell bodies are found in the midbrain (in the substantia nigra pars
compacta and in the ventral tegmental area). The cell bodies in the hypothalamus contribute to the
tuberoinfundibular pathway.
GABA (choice C) is the primary inhibitory neurotransmitter of the brain and is therefore located
throughout the brain.
Norepinephrine- (choice D) containing neurons are found predominantly in the locus ceruleus of the
pons and midbrain.

111
Q
  1. A boy is found to have multiple arteriovenous fistulas involving small cutaneous and
    subcutaneous arteries and veins of the right leg. Which of the following would be increased in
    a blood sample taken from the boy’s right femoral vein?
    A. Carbon dioxide content
    B. Hematocrit
    C. Oxygen content
    D. Plasma sodium concentration
    E. Total protein concentration
A

The correct answer is C. Congenital arteriovenous fistulas are often associated with limb swelling
and hypertrophy, visible pulsations when the fistulas are large, cosmetic changes when the fistulas
are in the subcutaneous tissues and skin, and varicose veins. The venous pressure is frequently
increased and the skin is often warmer compared with the opposite extremity. Because blood flowing
through the fistulas has by-passed the tissues of the extremity, the oxygen content of venous blood
from the involved limb is elevated as compared with the opposite limb, which is the pathognomonic
sign of arteriovenous fistula.
Because much of the femoral venous blood has bypassed the tissues, its carbon dioxide content
(choice A) is expected to be lower compared with venous blood of the opposite extremity (i.e., it
should be closer to that of arterial blood).
The venous hematocrit (choice B) is normally slightly greater compared with arterial hematocrit
because of a “chloride shift” into red blood cells as they pass through the microcirculation. This small
increase in hematocrit is attenuated with arteriovenous fistula as venous blood is diluted with arterial
blood that has bypassed the microcirculation.
There is no reason for sodium concentration (choice D) or total protein concentration (choice E) to
be affected by arteriovenous fistula, as these are normally similar in arterial and venous blood.

112
Q
  1. In the event of carbon monoxide poisoning, the blood sample taken from a systemic
    artery reveals
    A. normal hemoglobin level and normal oxygen content
    B. normal hemoglobin level with reduced oxygen content
    C. reduced hemoglobin level with reduced oxygen content
    D. increased hemoglobin level with increased oxygen content
A

The correct answer is B. Oxygen content is the total amount of oxygen carried in blood, that is, ml
oxygen/100 ml of blood. CO competes for oxygen binding sites on Hb, thus decreasing the oxygen
binding capacity of hemoglobin and decreasing the oxygen content of blood. Because CO does not
destroy or damage the hemoglobin molecule itself, the Hb level will remain normal despite the
reduction of oxygen carried by it. Choice A - Normal condition when no pathology is present. Choice
C - Hb concentration and oxygen content are decreased in anemia, as less hemoglobin is produced.
Choice D - Both Hb and oxygen content are increased in polycythemia.

113
Q
  1. An increase in which of the following is the most likely stimulus for the proliferation a
    large number of blood vessels in an abdominal tumor?
    A. Angiostatin
    B. Growth hormone
    C. Thrombospondin
    D. Vascular endothelial growth factor
    E. Tissue oxygen partial pressure
A

The correct answer is D. Vascular endothelial growth factor (VEGF) is a heparin-binding
glycoprotein that increases endothelial cell proliferation in vitro and increases capillary growth (i.e.,
angiogenesis) in vivo. Unlike most other growth factors, VEGF has unique target cell specificity for
vascular endothelial cells. VEGF is overexpressed in solid tumors and in ischemic areas of the heart
and retina. Levels are also reversibly increased in a variety of normal and transformed cells exposed
to a hypoxic environment. These characteristics of VEGF make it an ideal candidate as a regulator of
angiogenesis in physiologic and pathophysiologic situations in which vessel growth is preceded by
deficient perfusion of the tissues.
Angiostatin (choice A) is an antiangiogenic factor expressed by tumors that tends to inhibit
angiogenesis.
Growth hormone (choice B) has a general effect that causes growth of almost all tissues of the body,
but does not appear to stimulate angiogenesis to a significant extent in solid tumors.
Thrombospondin (choice C) is a multifunctional glycoprotein that interferes with tumor growth,
angiogenesis, and metastasis.
An increase in tissue oxygen partial pressure (choice E) is unlikely to be the stimulus for new vessel
growth because (1) solid tumors are invariably hypoxic or ischemic (i.e., the partial pressure of oxygen
is low), and (2) an increase in the partial pressure of oxygen in a tumor would tend to decrease the
expression of VEGF, thereby decreasing the amount of angiogenesis in the tissues.

114
Q
  1. Measurement of the blood pressure of a 65-year-old man reveals a systolic pressure of
    190 mm Hg and a diastolic pressure of 100 mm Hg. His pulse is 74/min and pulse pressure is
    90 mm Hg. A decrease in which of the following is the most likely explanation for the high
    pulse pressure?
    A. Arterial compliance
    B. Cardiac output
    C. Myocardial contractility
    D. Stroke volume
    E. Total peripheral resistance
A

The correct answer is A. A decrease in arterial compliance indicates that the arterial wall is stiffer
(i.e., less distensible). When the compliance of the arterial system decreases, the rise in arterial
pressure becomes greater for a given stroke volume pumped into the arteries. In the normal young
adult, the systolic blood pressure is approximately 120 mm Hg and the diastolic blood pressure is
approximately 80 mm Hg. Because the pulse pressure is the difference between the systolic and
diastolic blood pressures, the normal pulse pressure is approximately 40 mm Hg in a healthy young
adult. In older adults, however, the pulse pressure sometimes increases as much as two times normal
because the arteries become hardened by arteriosclerosis.
The cardiac output (choice B) itself has no direct effect on the pulse pressure; however, if a decrease
in cardiac output is associated with a decrease in stroke volume, the pulse pressure would be
expected to decrease. Cardiac output = stroke volume x pulse.
A decrease in myocardial contractility (choice C) would be expected to decrease stroke volume, and
therefore cause the pulse pressure to decrease. A decrease in stroke volume (choice D) causes the
pulse pressure to decrease because a smaller amount of blood enters the arterial system with each
heartbeat, and the increase and decrease of pressure during systole and diastole is decreased. This
would cause cardiac output to decrease. A decrease in total peripheral resistance (choice E) (i.e.,
vasodilation) does not have a significant effect on the pulse pressure of the major arteries under
normal conditions.

115
Q
  1. Which of the following hormones is most important in initiating gall bladder contraction?
    A. Cholecystokinin (CCK)
    B. Gastric inhibitory peptide (GIP)
    C. Gastrin
    D. Secretin
    E. Vasoactive intestinal polypeptide (VIP)
A

The correct answer is A. Cholecystokinin, or CCK, is synthesized in the duodenal and jejunal
mucosa and stimulates gall bladder contraction and pancreatic enzyme secretion. Other functions
include slowing of gastric emptying, an atrophic effect on the pancreas, and secretion of antral
somatostatin that in turn decreases gastric acid secretion. Gastric inhibitory peptide, or GIP (choice
B), stimulates pancreatic insulin secretion at physiologic doses and inhibits gastric acid secretion and
gastric motility at pharmacologic doses. Gastrin (choice C) prepares the stomach and small intestine
for food processing, including stimulating secretion of HCl, histamine, and pepsinogen. It also
increases gastric blood flow, lower esophageal sphincter tone, and gastric contractions. Secretin
(choice D) stimulates secretion of bicarbonate-containing fluid from the pancreas and biliary ducts.
Vasoactive intestinal polypeptide, or VIP (choice E), relaxes intestinal smooth muscle and stimulates
gut secretion of water and electrolytes.

116
Q
  1. What is the most common cause of an atrial septal congenital heart malformation?
    A. Failure of formation of the septum primum
    B. Failure of formation of the septum secundum
    C. Incomplete adhesion between the septum primum and septum secundum
    D. Malformation of the membranous interventricular septum
    E. Malformation of the muscular interventricular septum
A

The correct answer is C. The most common form of atrial septal defect is located near the foramen
ovale (not to be confused with a patent foramen ovale, which is of little or no hemodynamic
significance). They result from incomplete adhesion between the septum primum and the septum
secundum during development. Atrial septal defect is usually asymptomatic until middle age. Right
ventricular lift and S2 widely split and fixed is noted. A grade T-III/VI systolic ejection murmur at the
pulmonary area is often noted. Atrial septal defects less commonly result from failures of formation of
the septum primum (choice A) and septum secundum (choice B). Malformations of the
interventricular septum (choices D and E) cause ventricular septal defects rather than atrial septal
defects.

117
Q
117. Aspirin reduces fever and cause anticoagulation by inhibiting which of the following
pathways?
A. Phospholipase A2
B. Lipoxygenase
C. Prostacyclin
D. Cyclogenase
A
The correct answer is D. Aspirin and other NSAIDs inhibit cyclooxygenase, thereby inhibiting the
production of prostaglandins. This then decreases the set-point temperature. Inhibition of
phospholipase A2 (choice A) inhibits arachidonic acid. Inhibition of lipoxygenase (choice B) blocks
leukotrienes production. Blocking prostacyclin (choice C) does not decrease platelet aggregation.
118
Q
118. Which of the following directly inhibits insulin secretion?
A. Alpha2-adrenergic agonist
B. Beta2-adrenergic agonist
C. Cholecystokinin
D. Glucagon
E. Ingestion of a high-sugar meal
A

The correct answer is A. Alpha2-receptor agonists directly inhibit pancreatic insulin secretion. Beta2-
adrenergic agonists (choice B) stimulate insulin secretion. Cholecystokinin (choice C) is a hormone
that causes not only gallbladder contraction, but also insulin secretion from the pancreas. Pancreatic
glucagon release (choice D) acts as a paracrine stimulus for insulin secretion. Ingestion of high-sugar
meals (choice E) is a stimulus for the secretion of insulin from the pancreas.

119
Q
  1. Which of the following urinary constituents is probably present in abnormally high
    concentration and accounts for bubbles in the urine and generalized edema, especially
    noticeable in dependent regions and under the eyes?
    A. Albumin
    B. Chloride
    C. Glucose
    D. Sodium
    E. Urea
A

The correct answer is A. The combination of edema and frothy urine suggests nephrotic syndrome.
Urine with high protein content is more able to form stable bubbles than is normal urine, and may be a
clue that a patient (or parent) notices. The edema is caused by loss of albumin from the plasma,
lending to fluid loss from plasma to interstitial space. Chloride (choice B) in urine cannot be detected
grossly and would not cause frothy urine. Some people can detect a sweet smell to urine that contains
large amounts of glucose (choice C), and Greek physicians were known to taste urine to diagnose
diabetes, but the presentation suggests nephrotic syndrome. Sodium (choice D) in urine cannot be
detected grossly and would not cause frothy urine. Very concentrated urine with a high urea (choice
E) content looks darker in color and is not necessarily frothy.

120
Q
  1. Carbon dioxide is transported in blood by a variety of mechanisms. Which of the
    following is quantitatively the most important method for transporting CO2?
    A. As carbaminohemoglobin
    B. As CO2 in gas bubbles
    C. As CO2 in physical solution
    D. As sodium bicarbonate in red cells
    E. As sodium bicarbonate in serum
A

The correct answer is E. Red blood cells (and many other blood cells) contain the enzyme carbonic
anhydrase, which catalyzes the intracellular conversion of CO2 to bicarbonate and H+ ion. Most of the
bicarbonate in the red cell is exchanged across the plasmalemma for chloride ion. This means that
although the bulk of the production of bicarbonate occurs in the red cell (choice D), the bulk of the
actual transport occurs in serum. Carbonic anhydrase is not present in serum. Bicarbonate can be
produced in serum by nonenzymatic means, but the process is slow.
CO2 is also carried as carbaminohemoglobin (choice A), which forms when CO2 binds to an NH2
side group of the hemoglobin protein, rather than to the heme iron (Fe2+) as with carbon monoxide
and oxygen.
CO2 is not transported in the form of bubbles (choice B), which is a good thing, because gas bubbles
are effectively emboli, which can lead to considerable morbidity or death.
Some CO2 is carried directly dissolved in blood (choice C). It is 20 times more soluble in blood than
is O2.

121
Q
  1. Disease of which of the following structures would most likely cause a large difference in
    blood pressure between a patient’s legs and arms?
    A. Aortic valve
    B. Descending aorta
    C. Left atrium
    D. Left ventricle
    E. Mitral valve
A

The correct answer is B. A patient that has a large difference in blood pressure between his legs
and arms suggests disease of the aorta distal to the arch, where the vessels supplying the arms arise.
In younger individuals, coarctation of the aorta is the most probable diagnosis. In older individuals,
severe atherosclerosis of the abdominal aorta, iliac system, or femoral system is the most probable
diagnosis. Disease of the aortic valve (choice A), left atrium (choice C), left ventricle (choice D), or
mitral valve (choice E) would not produce selective effects on the blood pressure of the arms or legs.

122
Q
122. When a given substance is injected into an experimental animal, it increases intracellular
Ca+ through the formation of inositol 1,4,5-triphosphate. On which of the following receptors
does the given substance act?
A. Beta-1 receptor
B. Beta-2 receptor
C. Alpha-1 receptor
D. Alpha-2 receptor
E. M2 receptor
A

The correct answer is C. Phenylephrine acts on the alpha-1 receptor to increase intracellular Ca+
through formation of inositol 1,4,5-triphosphate. Isoproterenol acts on the beta-1 and beta-2 receptors
(choices A and B, respectively) through the activation of adenylate cyclase and production of cAMP.
Clonidine (choice D) acts on the alpha-2 receptor through the inhibition of adenylate cyclase and
decreases cAMP. ACH acts on the M2 receptor, which is inhibitory in the heart and decreases heart
rate and conduction velocity in AV node.

123
Q
123. In the event of propagation of an action potential, the conduction velocity is increased by
A. decrease in fiber size
B. increase in fiber size
C. unmyelinated fiber
D. increased internal resistance
A

The correct answer is B.Propagation of action potentials occurs by the spread of local currents to
adjacent areas of membrane that are then depolarized to threshold and generate action potentials.
Increasing fiber size increases conduction velocity. The greater the diameter, the greater the
conduction velocity. Choices A and D - Decreasing the diameter of nerve fiber results in increased
internal resistance, thus conduction velocity decreases. Choice C - Myelin acts as an insulator around
nerve axons and increases the conduction velocity. The greater the myelination, the greater the
conduction velocity.

124
Q
  1. During an experiment, the loss of a portion of the adrenal gland, which is controlled by
    the autonomic nervous system, results in a decreased capacity of the individual to mobilize
    glycogen or fat during exercise. Which of the following hormones of respective regions should
    be absent to match the outcome of this experiment?
    A. Epinephrine - medulla
    B. Epinephrine - zona reticularis
    C. Cortisol and androgen - zona reticularis and zona fasciculata
    D. Aldosterone - zona glomerulosa
    E. Aldosterone - Medulla
A

The correct answer is A.The major hormone of the adrenal medulla is epinephrine, a catecholamine
regulated by the autonomic nervous system. Its absence produces the above-mentioned effect.
Epinephrine is not a hormone of the cortex (choice B), and aldosterone is not a hormone of the
medulla (choice E). Cortisol and androgen (choice C), the hormones of the adrenal cortex, zona
fasciculata, and reticularis, are regulated by ACTH. Absence of cortisol could cause severe problems
under stressful conditions. Catecholamines do not exert vasoconstriction without cortisol, leading to
circulatory failure. Mineralocorticoid aldosterone, the hormone of the zona glomerulosa (choice D), is
under the control of angiotensin-2. Its absence could result in circulatory shock and death. Therefore,
it is necessary for survival.

125
Q
  1. Which of the following substances is most responsible for the constancy in plasma
    sodium concentration when large amounts of sodium are ingested?
    A. Aldosterone
    B. Angiotensin II
    C. Antidiuretic hormone (ADH)
    D. Atrial natriuretic factor (ANF)
    E. Epinephrine
A

The correct answer is C. A 5-fold increase in sodium intake causes the plasma sodium
concentration to increase by less than 1%, indicating the existence of a powerful mechanism for
48
maintaining extracellular sodium concentration at a constant level. When the ADH-thirst mechanism is
blocked, however, a 5-fold increase in sodium intake causes the plasma sodium concentration to
increase by more than 10%. The major mechanism for controlling extracellular sodium concentration
(and extracellular osmolarity) therefore is the ADH-thirst mechanism. You should recall that ADH
increases the permeability of the late distal tubule and collecting duct to water, which allows water to
be retained by the body and a concentrated urine to be excreted.
Aldosterone (choice A) and angiotensin II (choice B) are powerful salt-retaining hormones. They
regulate the total amount of sodium in the body, but have relatively little effect on plasma sodium
concentration under normal conditions for the following reasons: (1) they increase reabsorption of
sodium and water to an equal extent, and (2) any tendency for sodium concentration to change is
immediately compensated for by changes in ADH levels, which return sodium concentration to a
normal value.
Atrial natriuretic factor (choice D) is released from the atria when blood volume increases. It acts on
the kidneys to increase the excretion of sodium and water. ANF, however, does not play an important
role in regulating plasma sodium concentration because any tendency for sodium concentration (and
osmolarity) to change is immediately compensated for by changes in ADH levels, as discussed
previously.
Epinephrine (choice E) does not play an important role in regulating extracellular sodium
concentration.

126
Q
  1. To minimize chronic rejection injury to transplanted organs, immunosuppressive therapy
    is aimed at downregulating which of the following components of the immune response?
    A. Autoantibody production
    B. Complement protein synthesis
    C. HLA antigen expression
    D. Mast cell degranulation
    E. T-lymphocyte activity
A

The correct answer is E. Before any immune response can occur, such as a graft rejection, T cells
can be activated by exposure to an antigen. Chronic rejection of any solid organ entails cellular injury
to endothelial cells, resulting in intimal proliferation, fibrosis, and eventually ischemic injury to the
graft. Immunosuppressive therapy is directed at controlling lymphocyte activity and minimizing cellular
rejection.
Autoantibodies (choice A) are not involved in organ transplant rejection. The antibodies produced are
alloantibodies directed only to the graft, not to the host. Complement proteins (choice B) are involved
in the humoral component of acute rejection, and complement binding to alloantibodies increases
graft damage. Complement protein production, however, is not affected by immunosuppressive
therapy. HLA antigen expression (choice C) is central to recognition of foreign cells in grafted tissue.
HLA antigens are expressed constitutively by all normal cells, and immunosuppression does not affect
their production. Mast cell degranulation (choice D) is a component of the anaphylactic response
(Type I hypersensitivity). Graft rejection is a Type IV hypersensitivity response and does not involve
mast cell degranulation.

127
Q
  1. Which of the following laboratory parameters could be used to assess glycemic control
    over the past 2-3 months for a patient with type 2 diabetes?
    A. Blood glucose
    B. Blood insulin levels
    C. Blood ketones
    D. Glycosylated hemoglobin
    E. Urinary glucose
A

The correct answer is D. The range of normal for the HbA1c or glycosylated hemoglobin is 3.9-
6.9%. This is directly related to the level of glucose in the blood. Because HbA1c is a stable product,
its concentration reflects glucose levels over the past 2-3 months. HbA1c forms as a result of
nonenzymatic glycosylation, a fundamental biochemical abnormality that accounts for most of the
histopathologic alterations in diabetes mellitus. At first, glucose forms reversible glycosylation
products with proteins by formation of Schiff bases. Rearrangement of Schiff bases leads to more
49
stable, but still reversible, Amadori products and subsequently to irreversible advanced glycosylation
end products (AGE), of which HbA1c is an example.
Blood ketones, blood glucose, urinary glucose, and blood insulin do not reflect longstanding metabolic
abnormalities of diabetes mellitus and cannot be used to assess long-term glycemic control.
Blood glucose (choice A) is elevated in type 1 and type 2 diabetes mellitus. Hyperglycemia is the
diagnostic feature of diabetes mellitus and leads to glycosuria (choice E) when blood glucose
exceeds 160-180 mg/dL.
Blood insulin (choice B) is absent in untreated type 1 diabetes and normal or even slightly increased
in type 2 diabetes, depending on the state of the type 2 diabetes.
Blood ketones (choice C) (acetoacetic acid and beta-hydroxybutyric acid) are synthesized from free
fatty acids in response to severe insulin deficiency. Accumulation of ketone bodies in the blood is a
crucial pathogenetic factor in ketoacidosis, which occurs primarily in type 1 diabetics or type 2
diabetics with advanced disease who are improperly treated.

128
Q
  1. Daily serum samples from a woman with a normal menstrual cycle reveal decreasing
    progesterone and 17-estradiol levels. Serum LH and FSH levels are low, and begin rising.
    Basal body temperature begins falling. Within three days, which of the following events would
    be expected to occur?
    A. Markedly increased inhibin levels
    B. Menstruation
    C. Ovulation
    D. Rapidly decreased LH levels
    E. Significantly increased basal body temperature
A

The correct answer is B. A typical menstrual cycle lasts around 26-30 days. The luteal phase (postovulation)
generally lasts fourteen days; the length of the follicular phase (pre-ovulation) is far more
variable, and accounts for most of the variability observed in the length of the menstrual cycle. Just
before menstruation, sex steroid levels are low, but gonadotropin levels (especially FSH) begin rising
slightly. Basal body temperature remains high during the luteal phase of the menstrual cycle, but falls
precipitously a few days before the onset of menstruation. Markedly increased inhibin levels (choice
A) are seen in the middle of the luteal phase, dropping to low levels just before menstruation. LH
levels peak approximately 36 hours before ovulation (choice C), then decrease rapidly (choice D)
within a few days to a low level during the mid-luteal phase, gradually decreasing until menstruation.
The basal body temperature significantly increases (choice E) shortly after ovulation, due to the
metabolic effects of progesterone produced by the corpus luteum.

129
Q
129. Antigens processed by the exogenous antigen presentation pathway are presented in
association with which of following?
A. Fc receptors
B. IgG heavy chains
C. MHC class I molecules
D. MHC class II molecules
E. T cell receptor (TCR)
A

The correct answer is D. When pathogenic organisms are phagocytized and degraded in the
exogenous antigen presentation pathway, the antigenic molecules are presented on the surface of the
antigen-presenting cell by MHC class II molecules to a CD4+ T lymphocyte with a specific TCR for the
specific antigenic epitope. The Fc (choice A) portion of an antibody molecule is the part of the
immunoglobulin that attaches to the Fc receptors on phagocytic cell surfaces. When a Fab portion of
the antibody is attached to the pathogen and the Fc attaches to the phagocytic cell surface, the
phagocyte can destroy the pathogen more efficiently. The IgG molecule (choice B) is an
immunoglobulin that reacts with the antigen after it has been destroyed and presented to the T cell.
The IgG immunoglobulin is never involved in antigen presentation. In the endogenous antigen
presentation pathway (e.g., a virus infecting a cell), the cell would display epitopes from the virus in
association with class I molecules (choice C) to the CD8+ cytotoxic T cell. The TCR (choice E) is the
area of the mature T cell that reacts with the antigen epitope that is presented by the antigenpresenting
cell.

130
Q
  1. Most of the testosterone secreted by the testes exists in the plasma in the form of
    50
    A. dihydrotestosterone bound to gonadal steroid-binding hormone
    B. free dihydrotestosterone
    C. free testosterone
    D. testosterone bound to albumin
    E. testosterone bound to sex-steroid-binding globulin
A

The correct answer is E. Most circulating testosterone is bound to plasma protein (approximately
98%), rather than existing in free form (choice C). Of this, most is bound to a specific sex (or gonadal)
steroid-binding protein (choice E), and a minority is bound to albumin (choice D).
Dihydrotestosterone is produced from testosterone in the tissues by a specific enzyme, 5-alphareductase,
rather than circulating in bound (choice A) or free (choice B) form. Testosterone is the
principal androgen produced by the interstitial cells of the testes.

131
Q
  1. During a surgical procedure, cranial nerves IX and X are accidentally cut bilaterally. What
    would be the immediate change in the patient’s hemodynamic condition?
    A. Bradycardia with hypertension
    B. Bradycardia with hypotension
    C. Sinus arrhythmia with hypotension
    D. Tachycardia with hypertension
    E. Tachycardia with hypotension
A

The correct answer is D. The glossopharyngeal nerve (CN IX) and the vagus nerve (CN X) carry
afferent information to the medulla from the carotid sinus and aortic arch baroreceptors, respectively.
The firing rate of these neurons increases with increasing blood pressure. Severing these nerves
therefore sends the medulla a false signal that the patient has suddenly lost all blood pressure. This
elicits a baroreceptor reflex, resulting in an increase in sympathetic outflow and leading to tachycardia
and hypertension.
Remember physiologically the glossopharyngeal nerve routes to sensory nuclei of the medulla
oblongata. The somatic motor portion destination is the pharyngeal muscles involved in swallowing.
The visceral portion innervates the parotid salivary gland by way of the otic ganglion.
The vagus nerve routes to the sensory fibers to sensory nuclei and autonomic centers of the medulla
oblongata. The visceral fibers innervate the motor fibers and muscles of the palate, pharynx, and
digestive, respiratory, and cardiovascular systems in the thoracic and abdominal cavities.

132
Q
132. A patient with severe anemia may have which of the following symptoms?
A. Bradycardia
B. Cyanosis
C. Low stroke volume
D. Warm hands
E. Wide pulse pressure
A

The correct answer is E. The normal blood hemoglobin concentration is approximately 15-16 g/dL
for a man and 13-14 g/dL for a woman. A person is considered to be severely anemic when the
hemoglobin concentration decreases to less than 7.5 g/dL. In severely anemic, the resting cardiac
output is significantly increased with an increase in pulse and stroke volume (choice C). The increase
in stroke volume causes a widening of the pulse pressure, because when a greater amount of blood is
ejected during each systole, the blood pressure increases and decreases to a greater extent.
Bradycardia (choice A) is said to occur when the pulse decreases to less than 60/min. Severely
anemic exhibit tachycardia, which is defined as a pulse greater than 100/min. The increase in pulse is
because the body is trying to increase oxygen perfusion to tissues by increasing the heart rate.
Cyanosis (choice B) refers to a bluish color of the skin and mucous membranes that results from the
presence of deoxygenated hemoglobin in the blood vessels, especially the capillaries. Cyanosis does
not occur in severely anemic despite widespread hypoxia in the tissues because 5 grams of
deoxygenated hemoglobin must be present in each 100 mL of blood to produce overt cyanosis. In
other words, the hemoglobin concentration is too low for a severely anemic to become cyanotic. The
hands of anemic are often cold (choice D) because of decreased blood flow and oxygen perfusion to
the skin.

133
Q
  1. Radiographic studies of a 2-year-old child reveal a new fracture of the humerus and
    evidence of multiple old fractures in ribs and long bones of the extremities. Physical
    examination reveals that the toddler has “peculiar teeth,” a blue tinge to the sclera, and
    unusually mobile joints. The disease that the physician suspects the child has is characterized
    by an abnormality of which of the following biochemical functions?
    A. Collagen type I synthesis
    B. Collagen type II synthesis
    C. Collagen type III synthesis
    D. Collagen type IV synthesis
    E. Collagen type V synthesis
A

The correct answer is A. The child has the most common variant (type I) of osteogenesis
imperfecta, which is an autosomal dominant genetic defect in the synthesis of type I collagen,
because of decreased synthesis of the procollagen alpha1(1) amino acid chain. This defect (unlike
that of the perinatal, lethal, type II form of osteogenesis imperfecta) is compatible with survival, but
does cause skeletal fragility, dentinogenesis imperfecta (abnormal teeth), blue sclera, joint laxity, and
hearing impairment. Spontaneous fractures occur in utero and in childhood. Unfortunately, many
families of children with this defect have had their children removed because of “abuse,” only to find
that the broken bones continue in the new environment. Type I collagen is found in skin, bone,
tendons, and most other organs. This causes severe osteoporosis. Less severe mutations in type I
collagen are common, resulting in collagen disarray and predisposing to hypogonadal (e.g.,
menopausal) or idiopathic osteoporosis. Type II collagen (choice B) is found in cartilage and vitreous
humor. Type III collagen (choice C) is found in blood vessels, uterus, and skin. Type IV collagen
(choice D) makes basement membranes. Type V collagen (choice E) is a minor component of
interstitial tissues and blood vessels. There are also type VI-XI collagens, which are minor
constituents of various tissues.

134
Q
134. Which of the following neurotransmitters is most important for the induction of REM
sleep?
A. Acetylcholine
B. Dopamine
C. Epinephrine
D. Norepinephrine
E. Serotonin
A

The correct answer is A. Acetylcholine is the neurotransmitter of primary importance for the
induction of REM sleep. Some of the other neurotransmitters do function in sleep, but REM sleep can
occur in their absence. In the CNS, ACH synapses are throughout the brain and spinal cord. In the
PNS, ACH is found at the neuromuscular junctions, preganglionic synapses of the ANS, neuroeffector
junctions of parasympathetic division, and in the sympathetic division of the ANS. Dopamine (choice
B) is a neurotransmitter with a role in voluntary movement, mood, cognition, and regulation of
prolactin release. Epinephrine (choice C) is important in sympathetic nervous system responses. It is
also a CNS neurotransmitter. Norepinephrine (choice D) is important in sympathetic nervous system
responses. It is also a CNS neurotransmitter involved in attention, arousal, and mood. Serotonin
(choice E) is a CNS neurotransmitter that plays an important role in mood and sensation. In the
periphery, it is involved in vascular regulation and digestive function.

135
Q
  1. The primary metabolic effect of the principal hormone secreted by the alpha cells of the
    pancreas is
    A. augmentation of calcium deposition in bone
    B. increase of amino acid storage in the liver
    C. promotion of lipogenesis in liver and adipose tissue
    D. inhibition of gluconeogenesis
    E. stimulation of glycogenolysis
A

The correct answer is E. Glucagon is released from the alpha cells of the pancreas in response to
hypoglycemia and stimulates glycogenolysis to increase serum glucose. Augmented calcium
deposition in bone (choice A) is achieved by calcitonin, which is secreted by the C-cells in the thyroid
gland. Glucagon plays no role in calcium metabolism. Glucagon favors amino acid conversion to
glucose (gluconeogenesis) rather than storage in the liver (choice B). Insulin (which generally has
opposite effects of glucagon) promotes lipogenesis in the liver and in adipose tissue (choice C), and
also promotes glycogen synthesis. Glucagon stimulates gluconeogenesis (choice D).

136
Q
  1. Compensation for high altitude will result in which of the following physiologic changes?
    52
    A. Decreased production of erythropoietin
    B. Decreased 2,3-diphosphoglycerate (2,3-DPG)
    C. Increased renal excretion of H+ ions
    D. Increased renal excretion of HCO3-
    E. Pulmonary vasodilation
A

The correct answer is D. Compensation for high altitude includes an increase in the renal excretion
of bicarbonate. The diminished barometric pressure found at high altitude causes arterial hypoxia,
which is sensed by peripheral chemoreceptors. The ventilation rate increases, thereby causing a
respiratory alkalosis. The kidney then compensates by increasing the excretion of HCO3-.
Erythropoietin is increased, not decreased in chronic hypoxia and at high altitude (choice A).
Increased erythropoietin leads to an increased hematocrit.
Another adaptation to high altitude is increased 2,3-DPG (compare with choice B), which shifts the
oxygen dissociation curve to the right. This facilitates the release of O2 in the tissue.
High altitude leads to respiratory alkalosis. The renal compensation is a metabolic acidosis
characterized by decreased H+ excretion and increased HCO3- excretion. Respiratory acidosis is
normally compensated with a metabolic alkalosis that would include increases in H+ excretion
(choice C).
Pulmonary vasoconstriction, not vasodilation (choice E), occurs in response to alveolar hypoxia, such
as would occur at high altitudes.

137
Q
  1. A decrease in which of the following parameters would tend to increase the glomerular
    capillary hydrostatic pressure?
    A. Afferent arteriolar resistance
    B. Bowman’s capsular hydrostatic pressure
    C. Capillary filtration coefficient
    D. Efferent arteriolar resistance
    E. Plasma colloid osmotic pressure
A

The correct answer is A. The glomerular filtration rate is the rate of filtrate formation at the
glomerulus. It is commonly used to assess renal function. A decrease in the resistance of the afferent
arteriole (i.e., arteriolar dilation) directly increases glomerular capillary hydrostatic pressure by
lessening the decrease in blood pressure that normally occurs along the vasculature proximal to the
glomerulus. (Recall that the afferent arteriole is upstream from the glomerulus; the efferent arteriole is
downstream from the glomerulus.) The glomerular capillary hydrostatic pressure is the determinant of
glomerular filtration rate most subject to physiologic control.
Bowman’s capsular hydrostatic pressure (choice B), capillary filtration coefficient (choice C), and
plasma colloid osmotic pressure (choice E) are important determinants of GFR but they do not have
any direct effect to increase or decrease the glomerular capillary hydrostatic pressure.
A decrease in efferent arteriolar resistance (choice D) would tend to decrease the glomerular
capillary hydrostatic pressure because the efferent arteriole is downstream from the glomerular
capillaries.

138
Q
138. Exogenous therapy with which of the following hormones would be most likely to slow or
prevent osteoporosis?
A. Cortisol
B. Epinephrine
C. Estrogen
D. Thyroxine
E. Vasopressin
A

The correct answer is C. Estrogen replacement in postmenopausal women seems to play an
important role in preventing or limiting development of osteoporosis in postmenopausal women.
Osteoporosis can be asymptomatic or may involve severe backache and spontaneous fractures.
53
There is often a loss of height. Serum parathyroid hormone, calcium, phosphorus, and 25(OH)D2 are
often normal. Demineralization of the spine, hip, and pelvis are common.
Cortisol (choice A) excess, as in endogenous or exogenous Cushing syndrome, is a contributing
cause of osteoporosis. Increased cortisol will worsen the condition.
Epinephrine (choice B) concentrations seem to be unrelated to osteoporosis. Increased levels will
increase pulse and blood glucose level.
Thyroxine (choice D) excess (e.g., in thyrotoxicosis) may contribute to bone loss in some cases of
osteoporosis.
Vasopressin (choice E) concentrations seem to be unrelated to osteoporosis. Vasopressin or
antidiuretic hormone will decrease the amount of water loss from the kidneys.

139
Q
  1. A type 1 diabetic is noncompliant with his required insulin therapy and develops
    hyperglycemia. The release of which of the following intestinal hormones would be
    stimulated?
    A. Gastic inhibitory peptide (GIP)
    B. Gastrin
    C. Motilin
    D. Secretin
    E. Somatostatin
A

The correct answer is A. Gastric inhibitory peptide (GIP) is produced in the duodenal and jejunal
mucosa by K cells and is released in response to intraluminal glucose and fatty acids. GIP is
sometimes called glucose-dependent insulinotropic peptide because it stimulates pancreatic insulin
secretion in the presence of hypergylcemia. Note that although GIP release would be stimulated, the
hormone would not have a pronounced effect in this type 1 diabetic, whose pancreatic islet cells do
not produce adequate amounts of insulin.
Gastrin (choice B) is synthesized and stored primarily in the G cells of the stomach and TG cells of
the stomach and small intestine. The stimuli for gastrin secretion include increased vagal discharge,
digestive products, calcium salts, and gastric distention. Gastrin stimulates HCl secretion by parietal
cells, histamine release from enterochromaffin cells, pepsinogen secretion by chief cells, gastric blood
flow, and contraction of gastric circular smooth muscle. It has a trophic effect on gastric and small
intestinal mucosa and the pancreas, increases lower esphageal sphincter (LES) tone, and is a weak
stimulus for the secretion of pancreatic enzymes and bicarbonate.
Motilin (choice C) is produced in the M and enterochromaffin cells of the duodenum and jejunum.
Secretion occurs during fasting. Motilin acts to regulate the migrating myoelectric complex (MMC).
Secretin (choice D) is synthesized and stored in the S cells of the mucosa of the upper intestine.
Acidification of the duodenal mucosa and the presence of fat and protein degradation produced in the
duodenum stimulate its secretion. The main role of secretin is to stimulate bicarbonate secretion from
the pancreas and liver.
Somatostatin (choice E) is synthesized and stored in the D cells of the pancreatic islets, in the gastric
antrum, and throughout the intestine. It is also present in the hypothalamus. It inhibits the release of
gastrin, cholecystokinin (CCK), and most other gastrointestinal hormones. In brief, it shuts off the gut.
Somatostatin inhibits the release of glucagon by pancreatic alpha cells, as well as the release of
insulin by the pancreatic beta cells (of the islets of Langerhans).

140
Q
140. Which of the following is most likely to decrease in skeletal muscles during exercise?
A. Arteriolar resistance
B. Carbon dioxide concentration
C. Lactic acid concentration
D. Sympathetic nervous activity
E. Vascular conductance
A

The correct answer is A. The increase in muscle blood flow that occurs during exercise is caused by
dilation of the arterioles (i.e., decreased arteriolar resistance). In normal skeletal muscles, blood flow
can increase as much as 20-fold during strenuous exercise. Most of this increase in blood flow can be
attributed to the dilatory actions of metabolic factors (e.g., adenosine, lactic acid, carbon dioxide)
produced by the exercising muscles.
Exercise causes the concentration of carbon dioxide (choice B) and lactic acid (choice C) to increase
in the muscles. Mass discharge of the sympathetic nervous system (choice D) occurs throughout the
body during exercise, causing arterioles to constrict in most tissues. The arterioles in the exercising
muscles, however, are strongly dilated by vasodilator substances released from the muscles.
A decrease in vascular conductance (choice E) occurs when the vasculature is constricted.
Resistance and conductance are inversely related, so that a decrease in arteriolar resistance is
associated with an increase in arteriolar conductance.

141
Q
  1. When the pulse is greater than 200/min and the blood pressure is 75/40 mm Hg, what
    adjustments have probably occurred in the cardiac cycle?
    A. Diastolic time has decreased and systolic time has increased
    B. Diastolic time has decreased but systolic time has decreased more
    C. Systolic time has decreased and diastolic time has increased
    D. Systolic time has decreased but diastolic time has decreased more
    E. Systolic time has decreased but diastolic time has not changed
A

The correct answer is D. Under normal conditions, one third of the cardiac cycle is spent in systole
and two thirds spent in diastole. As pulse increases dramatically, the time spent in diastole falls
precipitously but the time spent in systole falls only slightly. A large increase in pulse must produce a
decrease in both diastole and systole (compare with choice A). The major change with increased
pulse is in diastole, not systole (compare with choice B). Pulse cannot increase if diastolic time
increases (choice C). An increase in pulse must be accompanied by a decrease in diastolic time
(compare with choice E).

142
Q
142. Which of the following is likely to be decreased in a patient with severe anemia (Hb less
than 7 g/dL)?
A. Arterial O2 content
B. Arterial O2 saturation
C. Arterial PO2
D. Cardiac output
E. Pulse
A

The correct answer is A. A decrease in the hemoglobin concentration of the blood causes a
proportional decrease in the oxygen-carrying capacity of the blood. Each gram of hemoglobin can
normally carry a total of 1.34 grams of oxygen. Thus, each 100 mL of arterial blood can normally carry
approximately 20 mL oxygen at a normal hemoglobin concentration of 15 g/dL blood. With a
hemoglobin concentration of 7 g/100 mL, each 100 mL of blood can carry only 9.4 mL oxygen. The
oxygen saturation of hemoglobin in the arterial blood (choice B) and the arterial PO2 (choice C) are
virtually unaffected by the hemoglobin concentration of the blood. The reduced oxygen-carrying
capacity of the severely anemic patient is associated with a compensatory increase in cardiac output
during resting conditions and especially during exercise. The elevation in cardiac output helps to
maintain oxygen delivery to the tissues at an adequate level. The increase in cardiac output (choice
D) is caused by an increase in pulse (choice E).

143
Q
143. Maximal ventricular Na+ channel conductance occurs during which phase of the ECG?
A. P wave
B. QRS interval
C. ST interval
D. T wave
E. U wave
A

The correct answer is B. Phase 0 of the cardiac muscle action potential (AP) corresponds to the
opening of voltage-dependent sodium channels, causing a transient but large increase in sodium
conductance during ventricular depolarization. The shape of the QRS complex of the ECG is
55
determined by the spread of the combined phase 0 (depolarization) of all the ventricular muscle of the
heart. The P wave (choice A) corresponds to atrial depolarization. The ST interval (choice C)
represents the time interval during which all ventricular cells are in phase 2 of their AP. Phase 2 is
dominated by a high, prolonged calcium conductance through slow channels. The length of the ST
interval corresponds closely to the AP duration in ventricular muscle. The T wave (choice D)
corresponds to ventricular repolarization. The U wave (choice E) is found only occasionally in ECGs
and is presumed to be caused by the repolarization of papillary muscle.

144
Q
144. During normal diastole, which of the following is most important in preventing overdistension
of the ventricles?
A. Adjacent lungs
B. Aortic valve
C. Diaphragm
D. Fibrous pericardium
E. Mitral valve
A
The correct answer is D. The fibrous pericardium, which surrounds the heart, does not simply
separate the heart from other chest structures, but has the important physiologic role of limiting the
distension of the heart during diastole. This helps keep the (normal) heart functioning in a useful part
of Starling's curve. In congestive heart failure, the slow enlargement of the heart also enlarges the
fibrous pericardium, and this protective function may be lost.
The lungs (choice A) and diaphragm (choice C) do not usually significantly limit cardiac expansion
during diastole. Shutting and opening of the aortic (choice B) and mitral valves (choice E) are
mechanical events that occur secondary to the changes in pressure in the cardiac chambers.
145
Q
  1. Physical examination of a pregnant woman in her second trimester reveals a slightly
    enlarged, nontender thyroid gland and a normal cardiac examination. The serum thyroxine (T4)
    level is increased; however, the serum thyroid-stimulating hormone (TSH) is normal. Which of
    the following best explains the laboratory findings in this patient?
    A. Decreased estrogen
    B. Increased free thyroxine (T4)
    C. Increased progesterone
    D. Increased serum triiodothyronine (T3)
    E. Increased thyroid-binding globulin (TBG)
A

The correct answer is E. The total serum thyroxine (T4) represents the sum of the T4 bound to
thyroid-binding globulin (TBG) and the free T4. An increased total T4 may be caused by an increase
in TBG or an increase in free T4, the latter leading to signs of thyrotoxicosis.
In a euthyroid state, one third of the binding sites on TBG are occupied by T4. An increase in estrogen
(pregnancy, birth control pills) increases the synthesis of TBG. The T4 bound to the additional TBG
increases the total serum T4. The extra TBG, however, does not alter the free T4 level because of the
equilibrium between the serum concentration of T4 and thyroid gland T4 production. Because the free
T4 level is normal, there is no stimulus to release thyroid-stimulating hormone (TSH) from the pituitary
gland. Regarding the patient’s enlarged thyroid gland, heat intolerance, and palpitations, these are
normal findings in pregnancy and do not indicate an overactive thyroid gland.
Estrogen is increased (not decreased) in pregnancy (choice A).
The serum free T4 (choice B) is normal in pregnancy. This explains why the serum TSH is normal in
the presence of an elevated serum T4, which reflects the increase in TBG normally occurring in
pregnancy.
The increase in progesterone (choice C) during pregnancy has no effect on TBG levels.
Although the serum T3 (choice D) concentration is increased in pregnancy for the same reason as
serum T4 (more T3 is bound to TBG), it is not responsible for the increase in synthesis of TBG that
leads to the increase in serum T4.

146
Q

146.Administration of an experimental drug that acts on peripheral nervous system (PNS)
myelin is shown to increase the space constant of an axon in a peripheral nerve. Action
potentials traveling down the axon would be predicted to be
A. faster
B. larger
C. slower
D. smaller
E. unchanged

A

The correct answer is A. The space constant of an axon reflects the amount of passive or
electrotonic spread of current within the axon. The larger the space constant, the further the current
can spread, allowing action potentials to propagate faster. This is why myelin increases the
conduction velocity of action potentials down an axon. Conversely, demyelination decreases the
space constant and slows action potential conduction.

147
Q
  1. Respiratory rate: 15/min; Arterial pressure: 120/80mm Hg; Cardiac output: 5L/min; Heart
    rate: 50/min Basal measurements are shown above. What is the stroke volume during resting
    conditions (in mL/min)?
    A. 50
    B. 75
    C. 100
    D. 125
    E. 150
A

The correct answer is C. The cardiac output (CO) is equal to the volume of blood ejected from the
heart during each beat of the heart (SV, stroke volume) multiplied by the number of times the heart
beats each minute (HR, heart rate). That is, CO = SV x HR. Therefore, SV = CO/HR, and since CO =
5000 mL/min, and HR = 50/min, SV = 5000/50 = 100 mL.
Note that some of the data listed are irrelevant to the solution of the problem

148
Q
  1. The drug used in the induction of anesthesia facilitates which of the following receptors
    action, which increases chloride conduction?
    A. GABA-B
    B. Glycine
    C. GABA-A
    D. Glutamate
A

The correct answer is C. Barbiturates facilitate GABA-A action by increasing the duration of chloridechannel
opening, thus decreasing neuron firing. GABA-B receptor (choice A) increases K+
conductance. Glycine (choice B) is an inhibitory neurotransmitter that increases Cl- conductance.
Glutamate (choice D) is an excitatory neurotransmitter in the brain. The kainite receptor for glutamate
is an ion channel for Na+ and K+.

149
Q
  1. A 45-year-old man with acromegaly has an MRI revealing the presence of a 1.5-cm tumor
    in the anterior pituitary. Which of the following endocrine abnormalities is likely to be present?
    A. Decreased plasma growth hormone concentration
    B. Decreased plasma IGF-1 concentration
    C. Decreased plasma insulin concentration
    D. Impaired glucose tolerance
A

The correct answer is D. The patient probably has acromegaly caused by a growth hormonesecreting
adenoma in the anterior pituitary. Hypersecretion of growth hormone in an adult will not
cause an increase in stature, because the epiphyses of long bones have already fused. Overgrowth of
bone in the face and skull, however, produces the characteristic protruding jaw and forehead
observed in this disorder. Soft tissue proliferation leads to a coarsening of facial features. The hands
and feet are particularly affected, producing large and thickened spade-like fingers and toes.
Excessive growth hormone decreases the sensitivity of peripheral tissues to insulin (“anti-insulin”
57
effect). This tends to raise blood glucose and produce a compensatory hyperinsulinemia (not
decreased plasma insulin, choice C) that functions to limit the hyperglycemia. Approximately 50% of
patients with acromegaly show impaired glucose tolerance.
Plasma levels of growth hormone (choice A) and IGF-1 (choice B) are both increased in acromegaly.

150
Q
  1. Which of the following is a characteristic of steroid hormones?
    A. Activation of adenylate cyclase
    B. Activation of protein kinases
    C. Plasma membrane receptors
    D. Stimulation of cellular protein synthesis
    E. Termination of effects by phosphodiesterase
A

The correct answer is D. Peptide/protein hormones and steroid hormones act by very different
mechanisms. Steroid hormones circulate in the bloodstream, then leave the bloodstream by
dissolving in the lipid-rich plasma membrane. Steroids cross the plasma membrane and enter the
cytoplasm, where they bind to a mobile protein receptor. The hormone/receptor complex then enters
the nucleus and triggers RNA synthesis, leading to protein synthesis, thereby changing the cellular
response. Hormones such as cortisol and aldosterone are steroid hormones.
In contrast, protein and amine hormones bind to specific receptors on the outer surface of the plasma
membrane (choice C), but do not enter the cytoplasm. Many activate adenylate cyclase (choice A)
and thereby increase cAMP. The increased cAMP stimulates protein kinases (choice B), which
change the target cell responses by phosphorylating intracellular proteins. Hydrolysis of cAMP by
phosphodiesterases (choice E) terminates the effect of the hormone.

151
Q
  1. Stimulating glossopharyngeal afferent fibers that supply the carotid sinus would most
    likely cause which of the following changes?
    A. Hypertension with bradycardia
    B. Hypertension with tachycardia
    C. Hypotension with bradycardia
    D. Hypotension with tachycardia
    E. No changes in blood pressure or heart rate
A

The correct answer is C. The glossopharyngeal nerve (CN IX) and the vagus nerve (CN X) carry
afferent information to the medulla from the carotid sinus and aortic arch baroreceptors, respectively.
The firing rate of these neurons increases with increasing blood pressure. Stimulation of the
glossopharyngeal nerve therefore sends the medulla a false signal that the animal has suddenly had
an increase in blood pressure. This elicits a baroreceptor reflex resulting in a decrease in sympathetic
outflow and an increase in parasympathetic outflow, leading to hypotension and bradycardia.

152
Q
  1. What is the role of the macrophage during antibody formation?
    A. Activation of cytotoxic CD8 T cells
    B. Delayed hypersensitivity reaction
    C. Lysis of virus-infected cells
    D. Processing antigen and presenting it to T helper CD4 cells
    E. Synthesis of immunoglobulin
A

The correct answer is D. The macrophage is a phagocytic cell of the monocyte-macrophage system.
The macrophage phagocytizes exogenous antigens (e.g., a bacterium), degrading the antigen into
small epitopes and presenting them, on MHC class II molecules on its surface, to CD4 T helper cells.
Macrophages do not activate cytotoxic CD8 T lymphocytes (choice A). The major activator of
cytotoxic CD8 T lymphocytes is IL-2 from CD4+ THl cells.
Delayed hypersensitivity reactions (choice B) are the results of CD4+ THl cells. These cells do not
produce antibody. They secrete gamma interferon and interleukin 2 (IL-2), stimulating more cells to
become involved in the delayed hypersensitivity reaction.
58
The cells that participate in lysis of virus infected cells (choice C) are cytotoxic CD8+ T lymphocytes
that react with MHC class I molecules containing epitopes of the virus from the infected cell.
Macrophages do not participate in this activity.
Macrophages never synthesize antibody (choice E). B cells initially produce antibody, then are
converted to plasma cells or memory B cells. The stimulus for the production of this antibody comes
from T helper cells that were stimulated by epitopes presented to them by macrophages.

153
Q
  1. Which of the following is most important for maintaining adequate cardiac output early in
    the course of exercise?
    A. Decreased cardiac index
    B. Decreased diastolic blood pressure
    C. Increased heart rate
    D. Increased stroke volume
    E. Increased systematic vascular resistance
A

The correct answer is D. At the beginning of aerobic (isotonic) exercise, increased stroke volume is
the most important adjustment for maintaining adequate cardiac output (CO = HR x SV) or cardiac
output = heart rate x stroke volume.
There is an increased, not decreased, cardiac index (choice A) during isotonic exercise.
Diastolic blood pressure usually remains unchanged during isotonic exercise; it is not decreased
(choice B). In contrast, systolic blood pressure usually rises during isotonic exercise.
Increased heart rate (choice C) becomes more important later in isotonic exercise (beyond 50% of
maximal work capacity).
There is decreased, not increased, systemic vascular resistance (choice E) during isotonic exercise.

154
Q
  1. Which of the following regions of the nephron is capable of the greatest level of sodium
    reabsorption?
    A. Collecting duct
    B. Distal convoluted tubule
    C. Proximal convoluted tubule
    D. Thick ascending limb of the loop of Henle
A

The correct answer is C. About 70% of the sodium entering the proximal convoluted tubule is
actively reabsorbed. Chloride and water follow passively.
The collecting duct (choice A) reabsorbs about 3% to 5% of the entering sodium, via aldosteroneregulated,
electrogenic Na+-channel pumps. Antidiuretic hormone (ADH) increases H2O permeability
and reabsorption in this region.
The distal convoluted tubule (choice B) reabsorbs about 5% of the entering sodium via Na+/K+/2Clcotransport.
This region is relatively impermeable to water.
The ascending limb of the loop of Henle (choice D) reabsorbs about 20% of the entering sodium via
Na+/K+/2Cl- cotransport. This region is relatively impermeable to water

155
Q
  1. An infant has a coarctation of the aorta that reduces renal blood flow to 50% lower than
    normal. Which of the following is increased in this infant?
    A. Blood flow in the lower body
    B. Glomerular filtration rate
    C. Plasma levels of angiotensin II
    D. Renal excretion of sodium
    E. Renal excretion of water
A

The correct answer is C. The lower than normal pressure at the level of the kidneys causes renin to
be secreted and angiotensin to be formed. Renin is released by the kidney because of (1)
59
sympathetic stimulation and (2) a decline in renal blood flow. When renin acts on angiotensinogen, a
plasma protein produced in the liver, it converts angiotensinogen to angiotensin I. Angiotensin I is
converted to angiotensin II by the enzyme ACE (angiotensin converting enzyme). The angiotensin
causes salt and water retention so that within a few days to weeks the arterial pressure in the lower
body (at the level of the kidneys) increases.
Blood flow in the lower body (choice A) is lower rather than increased, because of the blockage.
Blood flow, however, can be normal, above, and below the constriction if the body is able to
compensate fully.
The decrease in blood pressure at the level of the kidneys causes the glomerular filtration rate
(choice B) to decrease.
Increased plasma levels of angiotensin II cause salt and water retention; thus, salt and water
excretion (choices D and E) are decreased.

156
Q
  1. A patient with small-cell bronchogenic carcinoma of the lung complains of muscle
    weakness, fatigue, confusion, and weight gain. Serum sodium is found to be elevated and the
    tumor is secreting ectopic ADH. Which of the following abnormal laboratory results would also
    be expected?
    A. Decreased plasma atrial natriuretic peptide (ANP) concentration
    B. Decreased plasma vasopressin concentration
    C. Decreased serum osmolarity
    D. Decreased urinary sodium concentration
    E. Increased plasma aldosterone concentration
A

The correct answer is C. Bronchogenic carcinomas can secrete ectopic vasopressin (ADH), leading
to the syndrome of inappropriate ADH (SIADH). As long as water intake is not decreased, the
increased plasma vasopressin (not decreased, choice B) causes excessive water reabsorption by the
renal distal tubule and collecting duct. The increased total body water can explain the weight gain.
Edema is usually absent because the extra free water is distributed to intracellular and extracellular
volumes. The extra plasma water produces a dilutional hyponatremia, which can explain the
weakness, fatigue, and confusion. There will also be a dilutional decrease in serum osmolarity. With
SIADH, the urine sodium is usually increased (not decreased, choice D) compared with normal. This
leads to an inappropriately concentrated urine. The volume expansion resulting from the excessive
water retention may be responsible for the increased urinary sodium. Volume expansion would
increase plasma ANP (not decrease, choice A) and increase renal sodium excretion. The volume
expansion would also inhibit renin secretion from the kidney with subsequent decrease in plasma
aldosterone (not increase, choice E). Decreased plasma aldosterone would then allow for increased
renal excretion of sodium.

157
Q
  1. A decrease in which of the following parameters would tend to increase the glomerular
    capillary hydrostatic pressure?
    A. Afferent arteriolar resistance
    B. Bowman’s capsular hydrostatic pressure
    C. Capillary filtration coefficient
    D. Efferent arteriolar resistance
    E. Plasma colloid osmotic pressure
A

The correct answer is A. A decrease in the resistance of the afferent arteriole (i.e., arteriolar dilation)
directly increases glomerular capillary hydrostatic pressure by lessening the drop in blood pressure
that normally occurs along the vasculature proximal to the glomerulus. [Recall that the afferent
arteriole turns into the glomerulus; the efferent arteriole drains the glomerulus.] The glomerular
capillary hydrostatic pressure is the determinant of glomerular filtration rate most subject to
physiologic control.
Bowman’s capsular hydrostatic pressure (choice B), capillary filtration coefficient (choice C), and
plasma colloid osmotic pressure (choice E) are important determinants of GFR, but they do not have
any direct effect to increase or decrease the glomerular capillary hydrostatic pressure.
A decrease in efferent arteriolar resistance (choice D) would tend to decrease the glomerular
capillary hydrostatic pressure because it would allow blood to more easily flow out of the glomerulus.

158
Q
158. Which of the following lung volumes or capacities cannot be measured directly using
simple spirometry?
A. Expiratory reserve volume
B. Functional residual capacity
C. Inspiratory reserve volume
D. Tidal volume
E. Vital capacity
A

The correct answer is B. The functional residual capacity is the amount of air left in the lungs after a
normal expiration. Because this volume cannot be expired in its entirety, it cannot be measured by
spirometry. Essentially, lung volume that contains the residual volume, which is the amount of air
remaining after maximal expiration (e.g., functional residual capacity and total lung capacity), cannot
be measured by spirometry. These volumes can be determined using helium dilution techniques
coupled with spirometry or body plethysmography.
The expiratory reserve volume (choice A) is the volume of air that can be expired after expiration of a
tidal volume.
The inspiratory reserve volume (choice C) is the volume of air that can be inspired after inspiration of
a tidal volume.
Tidal volume (choice D) is the amount of air inspired or expired with each normal breath.
Vital capacity (choice E) is the volume of air expired after a maximal inspiration.

159
Q
  1. Which of the following would be expected to increase in response to hemorrhage
    resulting in loss of 600 mL of blood?
    A. Arteriolar diameter in skeletal muscle
    B. Sodium excretion
    C. Sympathetic nerve activity
    D. Vagal nerve activity
    E. Water excretion
A

The correct answer is C. The decrease in blood pressure caused by hemorrhage activates the
baroreceptor reflex, which tends to increase sympathetic nerve activity and decrease parasympathetic
(vagal) nerve activity (choice D). The increase in sympathetic nerve activity constricts arterioles in
skeletal muscle (choice A) and elsewhere in the body. The fact the patient has lost 600 mL blood and
yet her blood pressure has decreased only slightly from a normal value of approximately 120/80 mm
Hg may be attributed to the following compensatory responses: baroreceptor epinephrine released
from the adrenal medulla, formation of angiotensin II, formation of vasopressin, and the capillary fluid
shift mechanism.
Activation of the renin angiotensin system during hemorrhage also plays an important role in
maintaining blood pressure. Angiotensin II increases blood pressure acutely by constricting arterioles
throughout the body (choice A), and chronically by decreasing the renal excretion of salt (choice B)
and water (choice E). The decrease in salt and water excretion returns blood volume to a normal
value.

160
Q
  1. Which of the following areas of the adrenal gland would be expected to increase in
    activity in a patient subjected to salt restriction?
    A. Adrenal medulla
    B. Zona fasciculata of the adrenal cortex
    C. Zona glomerulosa of the adrenal cortex
    D. Zona reticularis of the adrenal cortex
A

The correct answer is C. This question requires you to equate salt restriction with an increased
synthesis of aldosterone (aldosterone promotes sodium reabsorption) and then to remember that
aldosterone is produced in the zona glomerulosa (the outermost layer) of the adrenal cortex.
61
The adrenal medulla (choice A) secretes catecholamines.
The zona fasciculata (choice B) is the middle layer of the adrenal cortex. It primarily secretes
glucocorticoids.
The zona reticularis (choice D) is the innermost layer of the adrenal cortex. It primarily secretes
androgens such as dehydroepiandrosterone (DHEA).

161
Q
  1. A middle-aged patient develops acromegaly. Which of the following pair of hormones
    normally regulates the hormone responsible for the symptoms seen with this disease?
    A. Dopamine and norepinephrine
    B. LH and hCG
    C. Prolactin and FSH
    D. Somatostatin and GHRH
    E. TSH and ACTH
A

The correct answer is D. Acromegaly is typically produced by a growth hormone-secreting pituitary
adenoma. Growth hormone synthesis is predominantly regulated by hypothalamic GHRH (growth
hormone releasing hormone), and its pulsatile secretion is predominantly regulated by hypothalamic
somatostatin. Note that the question asks for regulating hormones, not the hormone directly
responsible for the disease.
Dopamine and norepinephrine are catecholamines that regulate smooth muscle tone and cardiac
function (choice A).
Luteinizing hormone (LH) regulates sex steroid hormone production by testes and ovaries. Human
chorionic gonadotropin (hCG) is produced by the placenta and has actions similar to LH (choice B).
Prolactin regulates menstruation and lactation. Follicle stimulating hormone (FSH) regulates ovarian
and testicular function (choice C).
Thyroid stimulating hormone (TSH) regulates secretion of thyroid hormones. Adrenocorticotropin
(ACTH) regulates glucocorticoid secretion (choice E).

162
Q
  1. If the luminal diameter of the right renal artery is decreased by approximately 50%, which
    of the following is most likely increased?
    A. Afferent arteriolar resistance
    B. Glomerular filtration rate
    C. Glomerular hydrostatic pressure
    D. Interlobar artery pressure
    E. Secretion of renin
A

The correct answer is E. The decrease in renal artery diameter causes a reduction in arterial
pressure within the kidney, which results in an initial decrease in glomerular hydrostatic pressure
(choice C) and glomerular filtration rate (choice B). The decrease in glomerular filtration rate
decreases the amount of sodium chloride that is delivered to the macula densa; in turn, the
juxtaglomerular cells secrete renin, and angiotensin II is formed. The angiotensin then mainly
constricts the efferent arterioles, which increases glomerular hydrostatic pressure and glomerular
filtration rate back toward normal. This macula densa feedback mechanism also attempts to return
glomerular hydrostatic pressure (and therefore glomerular filtration rate) to a normal level by
decreasing afferent arteriolar resistance (choice A).
An obstruction of the renal artery would decrease blood pressure in the interlobar arteries (choice D).